100% found this document useful (1 vote)
1K views94 pages

A. Ectoderm. B. Mesoderm (Correct Answer) C. Endoderm D. All of The Above

The document discusses various topics related to trauma and fractures: 1) Bone originates from mesoderm. Immobilization of fractures should include both proximal and distal joints. Immobilization is the single most important factor in fracture healing. 2) Local complications of closed fractures do not include infection. For traumatic patients, the first step in management is to secure the airway. 3) Factors discussed include internal reduction considerations for fractures, cell types involved in bone remodeling, and fracture classifications.
Copyright
© © All Rights Reserved
We take content rights seriously. If you suspect this is your content, claim it here.
Available Formats
Download as PDF, TXT or read online on Scribd
100% found this document useful (1 vote)
1K views94 pages

A. Ectoderm. B. Mesoderm (Correct Answer) C. Endoderm D. All of The Above

The document discusses various topics related to trauma and fractures: 1) Bone originates from mesoderm. Immobilization of fractures should include both proximal and distal joints. Immobilization is the single most important factor in fracture healing. 2) Local complications of closed fractures do not include infection. For traumatic patients, the first step in management is to secure the airway. 3) Factors discussed include internal reduction considerations for fractures, cell types involved in bone remodeling, and fracture classifications.
Copyright
© © All Rights Reserved
We take content rights seriously. If you suspect this is your content, claim it here.
Available Formats
Download as PDF, TXT or read online on Scribd
You are on page 1/ 94

Trauma:

General:
Origin of bone is from

a. Ectoderm.
b. Mesoderm (Correct Answer)
c. Endoderm
d. All of the above

the expected time of internal complete remodeling of bone transplant is /


a. 6 ms
b. 1 year
c. 2 years
d. never
Immobilization is not required in fracture involving:
Scapula.
Wings of ilium.
Rib.
Proximal humerus in elderly.
All of the above. (Correct Answer)
Immobilization of fractures of long bones should include
Fractured bone only
Joint involved in the fracture
Proximal join
Both proximal and distal joints (Correct Answer)
Distal joint

The single most important factor in fracture healing is:


Correct bone alignment.
Accurate reduction.
Immobilization. (Correct Answer)
Organization of clot.

Local complications of closed fractures do not include:


Malunion
Non-union
Infection (Correct Answer)
Sudek's atrophy
Joint stiffness
1st step in management of a traumatic patient?
a. Secure airways

pt conscious with multiple trauma first step in management :


a - assess airway .
b - iv line .
c - endotracheal intubation .
d - blood transfusion .

1
-scenario unstable pelvic fracture ,hemodynamic unstable become stable after transfusion of
2 lite crystalloid ,u.s appear blood in abdomen ,pelvic and scrotal hematoattt
Immediate urethratomy,
ct scan,
observation,
catheterization to momitor fluid
Patient after accident, the left rib cage moves inward during
inspiration and outward during expiration:
a. Flial chest.

Which score uses GCS, respiration and systolic blood pressure


a.injury severity score
b.Revised trauma score
c. Organ Injury Score
d. Acute Physiology Score

Hypotension begins to develop after……….blood loss


a. 30-40%
b. 15-20%
c. 10-15%
d. 20-30%

A 32-year-old man sustains an iliac wing fracture and a contralateral femur fracture. Twelve
hours later he has shortness of breath with tachypnea, hypoxia, and confusion. A chest
radiograph is normal. What is the most likely diagnosis?
a.Fat emboli syndrome
b. Adult respiratory distress syndrome
c. Pulmonary embolus
d.Tension pneumothorax

Internal reduction is considered in presence of:


Reduction impossible to the achieved or maintained.
Healing is expected to be delayed.
Pathological fracture.
All of the above. (Correct Answer)

cells never show mitotic divisions


a) osteoclasts
b) osteoblasts
c) osteocytes
d) chondrocytes

The enzyme found in osteoclasts but not in osteoblasts is:


Alkaline phosphatase.
Acid phosphatase. (Correct Answer)
Elastase.
Cytochrome oxidase.

2
??Bone deformity with ligament tear
Fix ligament first
Fix bone first
Fix both at the same time
Evenually OA then deal with it

Gastillow type III should be irrigated by /


a. onelitre
b. 6 litres
c. 3 litres
d. 9 litres

The most severe growth disturbance results from which of the following types of epiphyseal
injuries:
Separation of the epiphysis at the metaphyseal side of the epiphyseal plate
Separation of the epiphysis with a triangular fragment of the metaphysic
Intra-articular fracture involving the articular cartilage epiphysis and epiphyseal plate
Intra-articular fracture extending from the joint surface through the epiphysis and epiphyseal
plate to the metaphysic
Crashing injuries compressing the epiphyseal plate without displacement (Correct Answer)

A greenstick fracture:
Occurs chiefly in the elderly.
Does not occur in children.
Is a spiral fracture- of tubular bone.
Is a fracture where part of the cortex is intact and part is crumpled or cracked. (Correct
Answer)

0-Salter Harris Fracture Affects Which Zone


1-Hypertrophic@
2-Proliferative
3-Reserve
4-2ndry Spongiosa

Thoruston Holland sign in Salter Harris grade


a) I
b) II
c) III
d) IV

3 – Torus fracture which zone:


a- hypertrophic.
b- Proliferation.
c- . Primary spongiosa (metaphysis) ossification. (True answer).

3
according to wolfs law when increase the bending force on bone there is
a-increase bone formation on med side
b- increase on lat side
c- increase on tension side
a= medial=compression

What is True of Sudeck's atrophy of hand:


Hand is painful and swollen.
There is increased blood flow to para-articular areas
Cervical sympathectomy may be of help.
All are true. (Correct Answer)

??Spiral fracture in children?


a. Open reduction and internal fixation.

Spiral fracture is due to:


Blunt trauma.
Axial compression.
Twist. (Correct Answer)
Direct impact.

to gain stability the distance from the proximal one of the 2 distal screws of the interlocking
nail and the fracture must be at least:
1-2cm 2-4cm 3-6cm 4-8cm

A Gigli saw is:


An electrically driven circular bone saw.
A pneumatically driven bone saw.
A short straight bone saw.
A long twisted wire bone saw. (Correct Answer)

ender nails is considered:


a. cephalo-medullary
b.centro-medullary
c.condylo-cephalic
d.interlocking

Grad B bending fracture ttt be :


.Compresion plate
. Neutral plate
. Bridge plate
. Butress plate

in buttress plate all is true except:


A. it negates ‫يلغى او يقاوم‬the shearing and compression force
b. used in metaphyseo-epiphyseal fracture
c.contouring is a must
d. it is anchoured on the most stable fragment and the fragment that it support

4
(a)

Buttress plate is used in:


a. diaphyseal fracture
b. metaphyseal fracture
c. metaphyseo-epiphyseal fracture
d. epiphyseo-diaphyseal fracture

What factor is most likely to increase the rigidity of an external fixation system?
a. Increased pin diameter
b. Increased pin number
c. Decreased pin separation
d. Decreased distance of the side bar to the bone

26 years old military man after walking for 36 km he suffered from pain in the left thigh and
did AP&Lateral x-ray on the hip and thigh and were normal what is the common missed
diagnosis?
A, quadriceps ms rupture
b. stress fracture
c, osteosarcoma
d. infection

pain and tenderness in pt 30 y male in mid foot , repetitive exercices : stress fracture .

Malunion of a fracture is
a fracture which unites in a position of deformity (Correct Answer)
delayed union of a fracture
non-union of a fracture
followed by pseudoarthrosis
due to tuberculosis

The term delayed union is employed when the fracture fails to unite within:
1.5 times the normal union time. (Correct Answer)
Twice the normal union time.
2.5 times the normal union time.
None of the above.

Non-union is common in fractures of the following bones except the:


Carpal scaphoid
Neck of the femur
Lower third of the tibia
Talus
Tuberosity of the fifth metatarsal (Correct Answer)

Non-union in closed fractures may due to any of the following except:


Inadequate immobilization
Interposition of soft parts
Impaired blood supply
Inpsction of the fragments (Correct Answer)
Wide separation of the fragments

5
Causes of gangrene after fracture in a limb do not include:
Direct crushing of the tissues
Injury to the main vessels
Tight plasters
Septic infection (Correct Answer)
Clostridial infection

Myositis Ossificans Since 6 months In Upper Thigh In Patient 66 Year With Ttt
1)Indomethacin75 Daily For 6 Weeks....
2)Irradiation...
3) Resection@

The correct ttt of traumatic myositis Ossificans is by:


Prolonged immobilization acute ttt by immoblization 3 w
Active exercises
Passive stretching and massage
Both A and B
Both B and C (Correct Answer)

Myositis ossificans commonly occurs around:


Shoulder.
Elbow. (Correct Answer)
Wrist.
Knee.

----- The humidity of the operative room should be:


1 10- 20
2 25 -35
3 40-50
????
RTA patient lnjury vscularity in proximal &middle third
below knee better than trans
Trans equal above
Above knee better trans
Below better above

photo of blackish color of 2,3 toes of foot after correction of clawing toe by k wire in 2 toes ttttt
(observation, remove wire, amputation, advance wire )

The following statements about provisional amputation for infective gangrene are true
except that it:

May be urgently needed to control infection and toxemia.


Should be made through the healthy limb above the infected area
should be done as low as possible to allow reamputation at the optimum level.
Should provide free drainage.
May be carried out by the guillotine or flap method without closure.

6
Spine:
Case Senario Patient In Rta More Than Half Muscles Motor Power > 3
What Asia Classification ?
Grad A
Grade B
Grade C
Grade D @

tear drop fracture in cervical spine is caused by:


a. flexion and compression mechanism
b. extension mechanism
c. hyperextension mechanism
d. compression mechanism
(a)

Upper limb:
last bone to ossify is
The xyphoid process in the sternum

Which of the following is true of shoulder joint?


Composed of only 2 joints
Anterior posterior gliding of scapula of never occurs
Acromioclavicular joint is more important
Allows flexion, rotation and abduction. (Correct Answer)

in AC dislocation the displacement of the clavicle occur when:


A. disruption in AC lig.
B. disruption in CoracoClavicular lig.
c.Disruption in Coraco acromial lig.
(b)

First bone "clavicle" to ossify at .......... Weeks intrauterine


-5
-7
-9
-12

Which is the commonest fracture in children? /


Fracture clavicle (Correct Answer)
Supracondylar fracture
Green stick fracture of lower end of radius
All of the above.

Common injury to baby is/ The most common bone fractured during birth
Fracture humerus.
Fracture clavicle. (Correct Answer)
Fracture
Fracture femur.

7
All are true regarding Clavicle except
First bone to ossify
No treatment required for fracture except rest
Ossifies in membrane
Break at mid-point (Correct Answer)

Concerning fracture of the shaft of the clavicle, it is Not true that it:
Is usually due to direct trauma (Correct Answer)
Commonly involves the middle third
Is often associated with overriding of fragments
Causes dropping and deformity of shoulder
Is usually treated by figure-of-eight bandage

Fracture of clavicle is commonest at


Junction of medial I/3"5 and lateral 2/3Rl
Junction of medial 2/3 and lateral l/3 (Correct Answer)
Midpoint
Scapular end

First bone to ossify in foetal life is:


Femur.
Tibia.
Clavicle. (Correct Answer)
Sternum.

Newborn with fracture mid clavicle what is true?/


a. Most cases cause serious complication.
b. Arm sling or figure 8 sling used.
c. Most patients heal without complications.

What is True of clavicle fracture:


Non-union is rare.
Malunion is of no functional significance.
Reduction even if achieved is difficult to maintain.
All are true. (Correct Answer)

"Figure of Eight" bandage used commonly in the fracture of:


Scapula
Humerus
Clavicle (Correct Answer)
Metacarpals

In treating a fractured clavicle in a 14 month old child, the best procedure is


Open reduction
Shoulder cast
Figure - of - eight bandage (Correct Answer)
Kirshner pin

8
A fracture of the midshaft of the clavicle is best
treated by
clavicle rings
a figure-of-eight bandage
open reduction and plating
an intramedullary nail
a broad arm sling and analgesics (Correct Answer)

A child with midclavicular fracture and overriding of


the fragments is best treated by:
Supine bed rest with interscapsular sandbag support
Open reduction and internal fixation
Figure-of-eight bandage (Correct Answer)
Closed reduction and plaster fixation
Manipulative reduction and abduction splint

Mid clavicle fracture :


Surgery is always indicated if fracture is displaced
Figure-8-dressing has better outcomes than simple sling
Figure-8-dressing is strongly indicated in patient with un-union risk
Both figure-8 and simple sling has similar outcomes (Correct Answer)

All are true regarding Clavicular fracture except;


May be caused by a fall on to the outstretched arm
Commonly occurs between the insertions of the coraco-Clavicular and the
costoClavicular ligaments
May jeopardize blood supply to the overlying skin
Usually requires careful reduction. (Correct Answer)

The most common complication of clavicle fracture is


Injury to brachial plexus
Malunuion (Correct Answer)
Stiffness of shoulder
Non union.

Treatment regarding fracture of clavicle is:


Most common complication is malunion (Correct Answer)
Occurs at the junction of medial I /3rd and lateral 2/3rds
Usually occurs due to fall on elbow
Comminuted fracture is common

x-ray fracture of the scapula and clavicle:


Fail shoulder
flail chest
float shoulder
scapothoracic dissociation

The joint most likely to have recurrent dislocation is:

9
Ankle.
Knee.
Shoulder. (Correct Answer)
Patella.

??military person fight argument came with pain direct anterior shoulder apprehension test
30 degree of elevation and abduction what is it:
lesser tuberosity fr
rotator cuff tears ??

The incorrect statement about anterior dislocation of the shoulder joint is that:
Shoulder loses its rounded contour & becomes flattened
The elbow is abducted from the side
All movements of the shoulder are limited and painful
The anterior and posterior folds of the axilla are elevated (Correct Answer)
The hand cannot be placed on the opposite shoulder (Duga's test)

Photo Hill Sachs Lesion

Duga's test is helpful in


Dislocation of hip.
Scaphoid fracture
Fracture neck of femur.
Anterior dislocation of shoulder (Correct Answer)

Luxatio erecta
Tear of the glenoidal labium.
Inferior dislocation of shoulder (Correct Answer)
Anterior dislocation of shoulder.
Defect in the humeral head

the most common complication in shoulder dislocation in young patient


Instability recurrence

Reduction Of Ant Shoulder Dislocation In Prone Position:


Stimpson

A patient with recurrent dislocation of shoulder presents to the hospital. The doctor tries to
abduct his arm and to extend the elbow and external rotation, but the patient doesn't allow
to do so. This test is called;/
Duga's test.
Hamilton's test,
Callway's test.
Apprehension test. (Correct Answer)

Following statement regarding dislocation of shoulder are true except


Head of humerus usually dislocates forward from shoulder joint

10
Injury is produced by forced extension & external rotation of abducted ami
In posterior dislocation, appearance of shoulder is not normal (Correct Answer)
None of the above.

The most common complication of dislocation of shoulder joint is


Injury to brachial plexus
Injury to circumflex nerve (Correct Answer)
Ruj)ture of supraspinatous muscle
Rupture of deltoid muscle

Hill-Sachs lesion in recurrent shoulder dislocation is


Injury to humeral head (Correct Answer)
Rupture of tendon of supraspinatus muscle
Avulsion of glenoidlabrum
None of the above.

Regarding Recurrent dislocation of the shoulder, which of the following is false?


All traumatic dislocations will be recurrent (Correct Answer)
Recurrent dislocation results when the capsule is stripped, not torn
The humeral head is always within the capsule
All of these.

Which is true regarding shoulder dislocation?


Posterior dislocation is often over-looked
Pain is severe in anterior dislocation
Radiography may be misleading in posterior dislocation
All of the above. (Correct Answer)

The position of arm in anterior dislocation of shoulder is


By the side (Correct Answer)
In abduction
In abduction
In external rotation

Anterior dislocation of shoulder is most commonly complicated by


Axillary artery injury
Circumflex nerve injury
Recurrent dislocation
Axillary nerve injury (Correct Answer)

All are related to recurrent shoulder dislocation except


Hill sachs defect
Bankart lesion
Lax capsule
Rotator cuff injury (Correct Answer)

In Recurrent Anterior dislocation of shoulder, the movements that causes dislocation is


Flexion and internal rotation
Abduction and external rotation (Correct Answer)

11
Abduction and internal rotation
Extension

Recurrent dislocation is most common in the shoulder joint. Which one of the following is
not an important cause for the same?
Tear of the anterior capsule of the shoulder
Associated fracture neck of the humerus (Correct Answer)
Tear of the glenoid labrum
Freedom of mobility in the shoulder.

Recurrent dislocation is least common in


Shoulder
Knee (Correct Answer)
Patella
None

Attitude in subcoracoid dislocation of shoulder includes


Adduction
Limb on side of body (Correct Answer)
Abduction
Elevation

In Anterior dislocation of the shoulder the nerve involved is


Radial nerve
Circumflex nerve (Correct Answer)
Ulnar nerve
Median nerve

Hill-Sachs lesion is associated with


Recurrent dislocation of shoulder (Correct Answer)
Recurrent dislocation of hip
Perthes' disease
Fracture neck offemur.

Bankart's lesion is seen in


Anterior border of head of humerus
Posterior border of head of -humerus
Anterior glonoid cavity (Correct Answer)
Posterior glenoid cavity

Recent dislocations of shoulder joint are best


treated by:
Hippocrates' method of closed reduction
Kocher's manipulation
Modified Milch's manoevre (Correct Answer)
Open reduction
Putti-platt's operation

12
Anterior dislocation of shoulder may be complicated by:
Brachial plexus injury.
Tear of rotator cuff.
Fracture head of humerus.
All of the above. (Correct Answer)

A 75 years old uncle fell on his outstretched hand and hurt his shoulder. This was treated
but he came back 4 days later with inability to raise his shoulder. What is the cause?
Axillary nerve damage
Musculocutaneous nerve damage
Rotator Cuff tear (Correct Answer)
Fracture of proximal humerus
Shoulder instability

Traumatic anterior dislocation of shoulder with sensory loss in lateral side of forearm and
weakness of flexion of elbow joint, most likely injured nerve is:/
Radial nerve
Ulnar nerve
Axillary nerve
Musculocutaneous nerve (Correct Answer)

About shoulder that is adducted and internally rotated (what


is the mechanism of dislocation
a. Anterior subclavicular
b. Anterior
c. Posterior

A patient with epilepsy came with Lt shoulder pain, on


examination flattened contour of the shoulder, fixed adduction
with internal rotation. your DX/
a. Inferior dislocation
b. subacromal posteroir Dislocation
c. subglenoid ant dislocation
d. subclavicle ant dislocation

Which of the following is not true about posterior dislocation of shoulder? /


recurrent dislocation can develop
reduction can be unstable
patients with reduced dislocation can have good function
clinical diagnosis is easy (Correct Answer)
axillary nerve injury is uncommon

What is not True about fracture surgical neck of humerus:


Occurs due to fall on outstretched hand.
Common to children. (Correct Answer)
Osteoporosis is an important risk factor.

13
Non-union is uncommon.

14
Old patient with chronic shoulder dislocation 6 months with bony defect more than 50% the
best treatment is:
a. Hemiarthroplasty
b. Total shoulder arthroplasty
c. Conservative
d. Repair of the defect

80 years old patient suffered from four part fracture head of the humerus the traetment is
A. hemiarthroplasty
b.total shoulder replacement
c. reduction and fixation
d. conservative treatment

fracture of the humerus related to which nerve injury


a. Radial nerve
(Axillary nerve injury in high fractures, radial in mid-fractures, and
median/ulnar nerve injuries in low-fractures).

adult with distal 1/3 fracture humerus and disruption in the distalradio-ulnar joint and
swelling and pain in the forearm treated by?
A. closed reduction and cast
b. open reduction and plate w screw
c. closed reduction and interlocking nail
d. open reduction and interlocking nail
(b???)

Malunion of distal radius & DURJ dislocation and after 6 weeks pt. had limitation of supination
&pronation ttt :
open reduction of druj?
corrective osteotomy of radius?

Commonest cause of wrist drop is


Intramuscular injection
Fracture humerus (Correct Answer)
Dislocation of elbow
Dislocation of shoulder

Radial nerve palsy may occur in fr of humerus involving:


Surgical neck.
Shaft. (Correct Answer)
Lower end.
At all of the above locations.

A boy fell down from a tree and has fracture neck of humerus. He cannot raise his arm
because of the involvement of:
Axillary nerve (Correct Answer)
Supraspinatus nerve
Musculocutaneous nerve
Radial nerve
15
Fractures of the shaft of the humerus are best treated by:
Closed reduction and shoulder spica
Continuous skeletal traction
Open reduction and internal fixation
Hanging plaster cast
Coaptation plaster splint with a Velpeau dressing (Correct Answer)

After reduction of humerus fracture distal third drop hand what do u call this injury
Holustein lewis
Thruston holland

Disadvantage Of The Im Nail Of The Humerus Comparing With Plate Is


1-Lower Reoperation Rate
2-Higher Union
3-Iatrogenic Communition of the Fracture
4-Elbow Pain

??Adult patient with # shaft humerus and closed reduction was done and ondoing the post
reduction x-ray there were 20 degree rotation and 15 degree medial angulation and 2.5 cm
shortenning we should do :
a.repeat the reduction
b. continue on the brace
c. open reduction and internal fixation
d. closed reduction and interlocking nail
)b(
criteria for acceptable
alignment include:
The most vulnerable structure in supracondylar
fracture of the humerus is the: < 20° anterior angulation
Median cubital vein
Brachial artery < 30° varus/valgus angulation
Median nerve (Correct Answer)
Ulnar nerve < 3 cm shortening
Radial nerve

Photo # Supracondylar Humerus Displaced Posteromedial


Which Nerve Affected :
. Ant Inter Osseou
. Post ..Inter
. Radial
. Ulnar

supracondylar g3 with absent pulse pink hand :


a -closed reduction and pulse check .
b - closed reduction pinning .
c –exploration .

16
The most common form of supracondylar fracture
humerus in children is of which type:
Flexion.
Extension. (Correct Answer)
Combination of A & B
None of the above.

Supracondylar fracture of the humerus in a child


is due to a fall on the point of the elbow
is usually compound
requires admission of the patient after reduction (Correct Answer)
requires immediate open reduction
is a fracture dislocation

Best management of SC fracture type III followed during reduction with absent pulse
a. retry reduction &check pulse
b. immediate surgical exploration
c. nerve conduction velocity test
d. intra operative angiography

A boy felt down on his elbow , the lateral x-ray shows:/


a. Anterior Pad sign
b. Posterior pad sign
c. Anterior line of humerous intersecting the cubilium
d. Radial line forming 90 degree with cubilium
the correct answer is B

Most serious complication of displaced supracondylarfracture humrus


1) compartement s
2) failure to heel
3) heel in non anatomical position
4) median nerve injury

Volkmann's contracture
affects the palmar fascia
develops at the ankle in a case of chronic venous ulcer
follows ischemia of the forearm muscles (Correct Answer)
is due to excessive scarring of the skin of the axilla following a burn
follows ulnar nerve palsy
Volkman's ischemia commonly occurs following:
Fracture shaft humerus.
Supracondylar fracture. (Correct Answer)
Colles' fracture.
Monteggia fracture.

The single dependable sign of early Volkmann's contracture is:


Cyanosis of fingers.
Obliteration of radial pulse.

17
Paralysis of flexor muscles of forearm.
Pallor of fingers.
Pain. (Correct Answer)

Cubitus valgus of elbow commonly follows fracture of:


Lateral condyle. (Correct Answer)
Medial condyle.
Capitalum.
Lower third of humerus.

A child is spinned around by holding his hand by his father. While doing this, the child
started crying and doesnot allow his father to touch his elbow. The diagnosis is:
Pulled elbow (Correct Answer)
Radial head dislocation
Annular ligament tear
Fracture olecranon process

coronoid # 65% with reduced and stable elbow


a.Cast& immobilization
b.Surgery ORIF
c. coronoid excision
d. bone graft

Terrible triad …
a . # head radius,#coronoid & elbow dislocation
b. readial head fracture, sc fracture &elbow dislocation
c. readial head fracture, medial condyle fracture &elbow dislocation
d. readial head fracture, lateral condyle fracture &elbow dislocation

X ray fracture neck radius 30 years ,intraoperative bone loss of metaphysis


Plate
Pinning
Excesion
arthroplasty

distal ulna fracture with limited supination routine x-ray after reduction not required what
next
1- CT for both wrist
2- x-ray for BOTH wrist
3- arthrography of both wrist
4- arthroscopy of the wrist

If we draw a line through the long axis of the radius it will


pass through the capitulum
a. Anterior pad signs
b. Posterior pad signs (sure)
18
If the line doesn’t pass through the capitulum = elbow dislocation.

The treatment of choice of fracture of radius and ulna in a an adult is


Plaster for 4 weeks
Closed reduction and calipers
Only plates (Correct Answer)
Kuntscher nails

Nerve At Risk In Prox Radius Fixation


PIN
AIN
Radial
Ulnar

Child with radial head dislocation, what is the next in


management:
a. Reduction with supination
b. X-ray
c. MRI

percent of articulation between proximal head radius and prox ulna ?


75%

9 ys child with dislocation of the head radius 5 months ago + pain and limitation of
movement of the elbow joint treated by
a. open reduction and annular ligament reconstruction
b. open reduction and recostruction of the lateral collateral lig.
c.open reduction and ulnar osteotomy (correct)
d. closed reduction and cast

A young adult presenting with oblique, displaced, comminuted fracture olecranon treatment
of choice.
1)ORIF Tension Band........
2)ORIF Plate ..
3)CR+long arm casting
4)fragment excesion+ advancement of tricepse tendon

A young adult presenting with oblique, displaced fracture olecranon treatment of choice;
Plaster cast.
Percutaneous wiring,
Tension band wiring.
Removal of displaced piece with triceps repair.

19
adult patient with fracture shaft ulna 6 months ago and treated with closed reduction and
cast now there is sill pain in the fracture site and x-ray show hypertrophic nonunion treated
by:
a. continue on the cast
b. fixation by plate without bone graft
c. fixation with plate and iliac crest bone graft
d. closed reduction and interlocking nail.
(b)

The best approach for midshaft radial fracture


a. Henry approach
b. Thompson approach
c. Kocher approach
d. Allis approach

Post ORIF radius and ulna synostosis occurred :


A – best is surgical removal of synostosis as soon as it happen
B –long screws in fixation are not a apt factor
C – post removal short wave radiotherapy helps to decrease recurrence
D – using single approach to fix both bones is apt factor
E – distal intra-articular synostosis gives best prognosis after removal
(D)

12 boy with # both bone forearm after 2 trial reduction under hamtoma block
Oppostioning 75% & 20 degree volar angulation :
.Observation
. Repeat reduction & pinning
. plating

pt fall down on fully extended hand what is the fracture :


a. colle's fracture

Colles' fracture can be complicated by late rupture of:


Extensor pollicis longus (Correct Answer)
Abductor pollicis longus.
Adductor pollicis longus.
Flexor pollicis longus

Colles' fracture is
a fracture of the clavicle
a fracture about the ankle joint
common in elderly women (Correct Answer)
a fracture of head radius
fracture of die scaphoid

20
21
Not a complication of Colles' fracture
Stiffness of wrist
Stiffness of shoulder
Carpal tunnel syndrome
Wrist drop (Correct Answer)

xray of comminuted distal radius in 35y


(opif, cr, crif, ex fixation)

smith fracture the postion of cast


1-pronation dorsal
2- supination dorsal
3-volar supination
4-volar pronation

Colles #, CR and BEC was done, 6 hours later patient came with swelling of fingers and
severe pain and parasthesia in the radial three fingers..management
a)cast loosening and recasting in neutral
b) ORIF with median nerve neurolysis
c) Carpal tunnel release
d) Forearm elevation

The deformity of wrist in Colles' fracture is:


Madelung's deformity.
Dinner fork deformity (Correct Answer)
Buttonaire deformity
None of the above.

8 years with # diatal radius of 4 weeks with dorsal angulation 15 degree


leave
corrective osteotomy , <9y. 30°
correction and cast
>9y. 20°
xray show non union proximal scaphoid fracture :
a- scaphoid fixation by screw and bone grafting . ( true answer )
b – scaphoid resection and wrist arthrodesis .
c – excision of proximal row .
d –cast .

Avascular necrosis of bone is most common in/


Scapula.
Scaphoid. (Correct Answer)
Calcaneus.
Cervical spine

22
Patient with pain in the anatomical snuffbox, he most likely
has:
a. Boxer's fracture
b. Colle's fracture
c. Scaphoid fracture

Scapholunate Space Is 5 Mlm Athelts Fell On His Hand


1-Pinnin and Repair Ligament
2-Open Reduction
3-Cast

The carpal bone most commonly fractured is:


Triquetrum.
Hamate.
Capitate.
Scaphoid. (Correct Answer)

carpal tunnel view is for


1-hook of hamate
2- scaphoid
3- lunate ..

Lunate Dislocation Photo

??athelts fell on his hand


1-pinning and repair ligament
2-open reduction
3-cast

Bennett's fracture is
reversed Colles' fracture
fracture of the scaphoid bone in the wrist
fracture of the radial styloid (chauffeur's fracture)
fracture dislocation of the first metacarpal (Correct Answer)
cause of mallet finger

lower limb:

The most common injury following pelvic fracture is


of:
Bladder
Urethra. (Correct Answer)
Rectum.
Vagina

23
Pelvic Inlet View Angle :
. 30 Down @
. 60 Down
. 40 Upword
…………

Rta Pt Hit At His Side X Ray # Superior Pupic Ramus :


A-P Compression
Avulsion Injury
L-C I @
L-C II

morel lavallee syndrome occure in


a.pelvic fracture
b.spine fracture
c.femoral shaft fracture
(a)

45-Hip Dislocation With One Piece # Classification Thompson–Epstein


I
Ii @
Iii
Iv

RTA with hip dislocation and shock so causes of shock is


a. Blood loss
b. Urethral injury
c. Neurogenic

Limb shortening with adduction and internal rotation


occurs in which type of hip dislocation:///
Anterior.
Posterior. (Correct Answer)
Central.
All of the above.

how much percent of AVN in posterior dislocation of the hip : 5-40%.

multiple spurs around the hip with no spur around greater trochanter

the grade is 0 1 2 3
spurs around hip and greater with more then 9 mm between them
2

24
Basocervical 10 Years Old Femoral Neck the Treatment Is
Traction……
Spica …..
Orif …..
Crif

Normal neck-shaft angle of femur is


90°.
120°. (Correct Answer)
150°.
170°.

Meyer's procedure is a method for treatment of


Recurrent shoulder dislocation
Habitual dislocation of patella
Congenital dislocation of hip
Fracture neck of femur (Correct Answer)

Nelaton's line joins anterior superior iliac spine to:


Xiphisternum.
Pubic tubercle
Ischial tuberosity (Correct Answer)
Ischial spine.

After a successful pinning of a hip fracture, a 60 year old grandmother complains on the
7th post-operative day of leg pain. You think that you should examine her further for the
one most likely complication
missed fracture of the fibula
deep venous thrombosis (Correct Answer)
anterior compartment syndrome
peripheral arterial occlusive disease
cellulitis

Avascular necrosis of head of femur occurs at


Trochanteric region.
Transcervical region.
Subchondral region(Correct Answer)
Subcapital region

Fracture femoral neck can be diagnosed from:


Limb shortening
External rotation
Abduction.
A + B. (Correct Answer)

The main blood supply to head femur :


.Lig. Teres
Ascending branch of lateral circumflex femoral
Medial circumflex femoral

25
. 4th. Not remember

7 years child main blood. Supply :


. Lig teres
. Epiphyseal blood supply
. Metaphyseal bl supply
. Synovium

femoral neck fracture in pt 60 y male complaining rheumatoid arthritis :


a- hemiarthroplasty .
b - total arthroplasty .
c- traction
d – DHS .

blind 75 years old bad medical history # neck femur


open osteosynthes
unipolar
bipolar
THR
(3)

X-ray periprosthetic femoral fracture vancouver b2 with good bone stock the treatment is :
a -rvision by long prosthesis .( true answer ) .
b- plate+graft. Revision long stem.
c- circlage traction .

30 years old male , bone scanning ( FNF ) . treatment :


a - DHS ..
b - 3 pining . ( true answer )
c - traction .

Tension Plate In Femur:


Medial Cortex
Lateral Cortex & Medial Cortex Intact @

causes of failure of DHS in comminuted trochanter


duo to failure of lateral trochanteric wall

Bryant's triangle helps to assess


 Fracture neck of femur
 Iliac crest displacement
 Trochanteric displacement (Correct Answer)
 None of the above

26
Following a stumble on stairs, a 70-year-old ♂ felt severe pain in the hip and could not stand
up. O/E, there was shortening of the limb, external rotation deformity and tender thickening
of the greater trochanter. X-ray examination revealed:
Intracapsular fracture of the neck of the femur
Pertochanteric fracture of the femur (Correct Answer)
Dislocation of hip
Fracture of acetabulum
Fracture of greater trochanter

Which of the following is not True of intertrochanteric


fracture of femur:
Limb shortening.
Malunion.
Avascular necrosis of femoral head (Correct Answer)
Internal fixation is preferred

The tip-apex distance is the sum of the distances from the tip of the lag screw to the apex of
the femoral head on both the AP and lateral radiographic views. The sum should be

 25 mm
 20 mm
 15 mm
 10 mm

traction 90 / 90 : sutrochanteric fracture and lesser trochanter with distal fragement


( ilioposois pull ) .

F 73 Y HTN SEVERLY OSTEOPOROTIC e previous C-SECTION+ appendictomy+colls


fracture 2times before e closed reduction .tonight felt down and X-RAY showed grade 4
subtrochantric fracture ....Treatement is
1) THA
2) DHS
3) HEMIARTHROPLASTY
4) GAMMA NAIL

??injury to thigh with fracure neck femur and fracture shaft femur ttt
Dhs eretrograde nail
cephalomedullary nail
cannulated screw eretrograde nail

Most common complication of fracture shaft femur


is:
Malunion.
Nonunion.

27
Knee stiffness (Correct Answer)
Fat embolism

Treatment of isolated fracture of femur

ORIF-
IMN
External fixation might be the choice if there is extensive soft tissue
injury; anyhow, the general rule is that the management is operative.

In an adult patient with a fracture of the shaft of the femur:


 No blood can be lost without obvious swelling.
 No blood can be lost without obvious bruising.
 Two liters of blood can be lost without obvious swelling or bruising. (Correct
Answer)
 There is no possibility of death from hemorrhagic shock.

Brooker classification for HO


multible spurs around the hip with no spur around greater trochanter:
the grade is 0 1 2 3

spurs around hip and greater with more then 9 mm between them
2

female pt , with RTA, she has bilateral femur fracture, in this


scenario, systolic blood pressure 70, what will you do:
a. Iv fluid
b. blood transfusion

Femur shortening in which age group = 2cm accepted

a. 3-6 months
b. 6-18 months
c. 1.5-4 years
d. 1-3 months

In the following types of fractures of long bones,


crepitus can be elicited only in:
Fissures
Subperiosteal cracks
Greenstick fractures
Spiral and oblique fractures (Correct Answer)
Impacted fractures

The signs of fractured shaft of a bone do not include:

28
Swelling
Deformity
Loss of all movements in the limb (Correct Answer)
Acute localized bone tenderness
Abnormal mobility in the line of the bone

-fracture distal femure ttt 2y ago now child appear shortening and deformity ,radiograph of
knee to detect difference in length
articular join,
scanogram,

Fractures which do not impact include


fracture of tile vault of the skull
a compression fracture
a simple fracture
a transverse fracture of the patella (Correct Answer)
fracture of the neck of the femur

Treatment of a severe comminuted fracture of the patella includes


physiotherapy alone
insertion of a figure-of-eight tension band
patellectomy (Correct Answer)
inserting screws or wire
skin traction

photo : intra operative T shaped distal femur fracture


What is next step :
a - Restore articular surface
b - Restore metaphysis .

old pt have swollen knees and patella ballotment and fluid +ve ,,, what is the next step
a. MRI
b. X RAY
c. INCISION AND DRAINAGE
d. ???

photo X Ray Knee Fracture (As I See) Tibial Eminece What Next Step
.Traction
Post Slab ??
Immediate Physiotheraby
Aspirat

indication of open reduction and internal fixation in tibial platuea fracture when the
displacement in the articular surface is:
a. 2mm

29
b. 5mm
c.10mm
d. 15mm
(b)

the most common complication of ilizarov in treatment of comminuted fracture tibialplatuea


is
a. knee stiffness
b. pin tract infection
c. septic arthritis
b))

non union infected tibia with shortening :


a- unilateral ex fix .
b – p o p with reduction .
c- ilizarove ( true answer ) .

%?Of Knee Pain After Intramedullary Nail Tibia


-Less 20%
-50% @
-80%
-100%

interlocking tibia from 3 month now there is discharge from proximal locking screw ,xray
screw in its place and bone uniting ttt
remove nail
remove and ex fix
remove and cast
wait until united then remove

A pt presented with open tibial fracture II. Which antibiotic you will give?
a. Cefazolin
b. Gentamycin
c. Cefazolin & gentamicin
d. Cefazolin, gentamycin & metronidazole??

Union of a simple uncomplicated transverse fracture


of the tibia in an adult normally takes
6 weeks
8 weeks
12 weeks (Correct Answer)
18 weeks
26 weeks

A Pott's fracture is a type of fracture of the


wrist
ankle (Correct Answer)

30
spine
foot
skull

syndesmosis screw
self cortical taping
partial cortical threaded self tapping
cancellus screw partial threaded

syndesmotic screw inserted in the ankle in position :


a – 15 degree anterior and dorsiflexion of ankle .
b - 30 degree anterior and planterflexion of ankle .
c- 15 degree anterior and planterflexion of ankle .
d - 30 degree anterior and dorsiflexion of ankle .

Syndesmosis Screw Distance Plafond


1) More Than 4.5 Cm
2) 4 Cm
3)3 Cm
4) 2 Cm

in supination adduction lesion of ankle what most proper plating utilized in mm fracture
treatement
1.tension band
2.antiglide plate
3. bridging plate
4.neutral plate

lateral malleolus fracture transverse, medial malleolus fracture verticle by


pronaion abduction
supination adduction
pronation ex rotation
supination ext rotation

Male patient presents to ttt of tri malleolar fracture come to hospital 5 days after trauma.
Noo oedema. Xray shows posterior malleolar # less than 25%
Orif
Closed reduction +casting
Closed rductio+per cuyaneous pininig

In trimalleolar fracture which structure should be fixed first

31
a. Fibular
b. Medial malleolus
c. Posterior tibial fracture
d. Syndethmotic ligament

Syndesmotic ligament is MOST likely to be teared in

a.Weble type 1
b.Weble type 2
c.Weble type 3
d.Weble type 4

most common injury of ankle ligament


atfl
atifl
syndesmosis

photo female patient . tendernss on the lat malleolus tip and instability and second photo
anterior drawer of the ankle . the procedure :
 Brostorme operation
 Gould modification of Brostrom anatomic reconstruction
o procedure
 an anatomic shortening and reinsertion of the ATFL and CFL
 reinforced with inferior extensor retinaculum and distal fibular periosteum
o results
 good to excellent results in 90%
 consider arthroscopic evaluation prior to reconstruction for intra-articular evaluation
 Tendon transfer and tenodesis (Watson-Jones, Chrisman-Snook, Colville, Evans)
o procedure
 a nonanatomic reconstruction using a tendon transfer
o technique
 any malalignment must be corrected to achieve success during a lateral ligament
reconstruction
 Coleman block testing used to distinguish between fixed and flexible hindfoot
varus
o results
 subtalar stiffness is a common complication

What is the most common item cause long standing complication secondary to its fracture
a. talus
b. humerus
c. Malunion
d. Infection

according to Hawkin classification fracture talus + dislocation in ankle joint is /


a.type 1
b. type 2
c. type 3
d. type 4

32
(c)

33
Hawkin sign:

1. Radiolucent area at the neck of the talus


2. Avascular necrosis of the talus
3. Malunited talus
4. Ununited talus

fracture neck talus and fixed by screws and after 8 weeks there is radiolucent area appeared
near the dome what is the cause?
a.non union
b. infection
c. good revascularization
(c)

In which type of fracture, the tuber-joint angle is reduced to about half


Crush fracture of calcaneum
Fracture neck of humerus
Dislocation of shoulder
Spilt fracture of calcaneum (Correct Answer)??
Fracture neck of femur.

A patient had hairline metatarsal fracture. The x-ray was


normal. What is the 2nd line?
a. CT scan
b. MRI
c. US

General Ortho:

2-infection:
Bone marrow labelled with leucocytes under fluoroscopy is seen as
a) not visible
b) faint line
c) double faint line
d) mild opacity
?? a

A case of osteomyelitis, organism enters through?


a. Epiphysis
b. Metaphysis a
c. Nutrient artery (however, the most affected site is metaphysis)
d. Cortex of bone

The common unusual organism cause osteomylitis in drug abusers is


a. staph. Aureus
b. strep.Coccus

34
c. pseudomonus arogenusa )c(

Acute osteomyelitis is commonly caused by:/


Staph aureus. (Correct Answer)
S. pyogenes.
H. influenzae.
Salmonella.

54 -Which antibiotic is for choice in hospital acquired staph :


a - methecillin
b -vancomycin
c -gentamycin
d –Vancomycin ( true answer ) .for MRSA

-child 7y sceario case of osteomylities come to hospital immediate step


(immobilize, mri, cbc, esr )

What is True of acute pyogenic osteomyelitis:


Trauma is a predisposing factor.
Common infecting agent is Staph. Aureus.
Infection is usually blood borne.
All are true. (Correct Answer)

Acute osteomyelitis usually begins at:


Epiphysis.
Metaphysis. (Correct Answer)
Diaphysis.
Any of the above.

Metaphyseal Infection in 4 Y Old Spread:


Through Cortex to the Periosteum
through Epiphysis to Joint

pt has sickle cell anemia


what the investigation to defferentiate betweem osteomylitis and crisis of hemolytic about acute
hip pain????
1) CT
2)ESR
3)CRP bone scan defranciate infarction from infection
4)MRI e Gadolinium. Low uptake in infarction. High in infection

Sequestrum is
A piece of infected bone
A Piece of dead bone (Correct Answer)
Organised inflammatory exudates
Segregated marrow tissue

The sequestrum in X-ray appears:

35
Dense. (Correct Answer)
Light.
Isodense as surrounding bone.
Any of the above.

Seqestrum is
Dead piece of bone
Avascular part of bone
Deadbon surronded by healthy bone
deadbone surrounded by mature bone

What is not True of Brodie'sabscess:


A form of chronic osteomyelitis.
Intermittent pain and swelling.
Common to diaphysis. (Correct Answer)
Excision is very often required.

Gledhill classsification is based on


*location and
location and radiologz

9-year-old boy presented with limping and pain in


the right knee two days after a fall in the street. On
examination he looked ill and in severe pain with high
fever and swelling of the knee region extending to the
thigh which was warm and very tender. The most
probable diagnosis is:
Traumatic synovitis
Hemarthrosis
Acute osteomyelitis of the femur
Septic arthritis of knee
Bone sarcoma

knee aspiration result 200 WBCS 25 % PMNS SO:


.NORMAL
.RH ARTHRITIS
.SEPTIC ARTHRITIS
.TRAUMATIC SYNOVITIS

Tuberculosis of the spine most likely originates from:


Intervertebral disk.
Cancellous vertebral body. (Correct Answer)
Ligamentous structures.
Paravertebral soft tissue.

In Pott's spine, the disease starts in the:


Intervertebral disk.
Anterior vertebral margin.(Correct Answer)
Posterior vertebral margin.
Paravertebral soft tissue.

36
Melon seed bodies in joint fluid are characteristic of:
Rheumatoid arthritis.
Tuberculous arthritis. (Correct Answer)
Septic arthritis.
None of the above.

The earliest sign of TB hip in X-ray is:


Narrow joint space.
Irregular moth eaten femoral head.
Periarticular osteoporosis. (Correct Answer)
Dislocation.

Healing of tuberculous arthritis can lead to:


Calcification.
Fibrous ankylosis. (Correct Answer)
Bonyankylosis.
None of the above.

TB hip joint in external and abduction position what is the cause :


Synovitis
destruction of the femoral head . (FADIR+ shortining)

Cold abscess is commonly due to infection of


Skull
Ribs
Spine
Sternum

Actinomycosis is commonly seen in


Tibia.
Mandible (Correct Answer)
Scapula
Femur.

Arthritis of tertiary syphilis most frequently involves:


Shoulder joint
Elbow joint
Knee joint (Correct Answer)
All of these.

3-Rheumatoid:
Mechanism of destruction of joint in RA:
a. Swelling of synovial fluid

37
b. anti inflamtory cytokines attacking the joint (an abnormal immune
response triggering joint destruction).

Young male with morning stiffness at back relieved with activity and uveitis:
a) Ankylosing Spondylitis
B)?

Old patient treated from rheumatoid arthritis for a long period he came to your clinic suffers
from burning pain and numbness in the thumb and the next two fingers the diagnosis is
A. carpal tunnel syndrome
B. ulnar nerve injury
(a)
54. In a patient with rheumatoid arthritis:
a. Cold application over joint is good
b. Exercise will decrease post-inflammatory contractures

Bumboo Spine
Ankylosing Spondylitis

46- Typical case on ankylosing spondylitis ask about Rx: .?


NSAIDS – Analgesics – DMARDs – TNF – alpha

Ankylosing spondylitis
1-HLA-B27
2-HLA-DR4
3- ANA

74 years old female complaining of pain and stiffness in the hip and shoulder girdle
muscles. She is also experiencing low grade fever and has depression. On examination
no muscle weakness detected (Polymyalgia rheumatic). Investigation of choice:
RF
Muscle CK
ESR (Correct Answer)

4-Crystals
A man who is having a severe pain on his big toe with knee
pain and examination revealed negative perfringens crystals:
a. Uric acid deposit secondary to synovial fluid over saturation
b. Calcium pyrophosphate secondary to synovial fluid over saturation

Pt. with recurrent gout what you will give him

38
a. Allopurinol

"Gouty arthritis" usually involves first:


Ankle
Great toe (Correct Answer)
Thumb
Shoulder joint.
Old woman drunken of alcohol... get up in the morning with severe pain and inflammation of her
big toe ... aspiration is expected to show:
1 pyrophosphate
2 urate crystals
3 cacium phosphate
4 ???

5-Osteoarthritis:

malalignment of patellofemoral x-ray


1- lateral
2- AP
3- tunnel
4- Tomography. Axial xray

cartilage best bear load if it is loaded


a.Vertically
b. Cylicaly
c. Perpendicular
(b??c)

Question about OA radiographic signs.

The most common site of primary osteoarthrosis is


Hip joint
Knee joint (Correct Answer)
Ankle joint
Shoulder joint

Tom Smith arthritis manifests as;


Increase hip mobility and unstability (Correct Answer)
Hip Stiffness.
A+ B.
Shortening of limb

An old woman complaining of hip pain that increases by


walking and is peaks by the end of the day and keeps her awake
at night, also morning stiffness:

39
a. Osteoporosis
b. Osteoarthritis

An elderly lady presented with Swelling knee pain bilaterally


that increases with activity & decreases with no history of
trauma. The most likely diagnosis is:
a. Osteoarthritis

The useful exercise for osteoarthritis in old age to maintain


muscle and bone:
a. Low resistance and high repetition weight training
b. Conditioning and low repetition weight training
c. Walking and weight exercise

The best non-medical therapy is proven to be of benefit for osteoarthritis is:


a) Muscle strength exercise
b) Give NSAID
c) Back slap
d) ?
The correct answer is a

Rt. Hip OA /
Use the cane at Rt. or Lt.
(Lt)

Non medical TTT of osteoarthritis:


a. Muscle exercise.
b. Spine manipulation.
c. Analgesic cream local.

Best way to decrease pain in elderly with bilateral knee pain


and crepitation is:
a. NSAID
b. Decrease weight
c. Exercise

Diet supplement for osteoarthritis


a. Ginger

A patient present with long time history of knee pain suggestive of osteoarthritis.
Now he complains of unilateral Lower limb swelling and on examination there is
+ve pedal & Tibial pitting edema. What is the next appropriate investigation?
a. CXR

40
b. ECG
c. Echocardiography
d. Duplex ultrasound of lower limb (immobility can lead to DVT)

Best way to decrease pain in elderly with bilateral knee pain and crepitation is:
a) NSAID
b) Decrease weight
c) Exercise
d) ?
The correct answer is b , (I'm not sure )

What is the initial management for a patient newly diagnosed knee osteoarthritis.
a. Intra-articular corticosteroid
b. Reduce weight
c. Exercise
d. Strengthening of quadriceps muscle.
The correct answer is d

Old patient suffered from polyarticular arthritis and arthritis in the DIP with no
morning stiffness
a. rheumatoid arthritis
b. osteoarthritis
c. infection
d. TB
(b)

Spontaneous bleeding into joints in haemophilia occurs when factor Viii level is less
than:
50%.
25%.
10%.
5%. (Correct Answer)

Severe disability in primary osteoarthritis of hip is best managed by:


Arthrodesis
Arthroplasty (Correct Answer)
Mc Murray's osteotomy
Intra-articular hydrocortisone and physiotherapy.

THA, which quadrants liable in acetabulum in danger e screws ….


a. Post. sup. & post.inf.
B. ant superior & ant inferior
C. post inferior &ant superior
D. post superior & ant superior

The safe zone of THA is

41
Posterior superior + posterior inferior.
Anterior superior +anterior inferior
another mixed chices

THA When We Do Advancement Of The Greater Trochanteric Non-union Percept Is


1)5%....
2)10%...
3)20%....
4)30%

X-ray periprosthetic femoral fracture vancouver b2 with good bone stock the treatment
is:
a - rvision by long prosthesis . ( true answer ) .
b- plate+graft. Revision long stem.
c- circlage traction .

in THA of acetabularprotrusio the most artery affected is /


1-obturator
2-common illiac
3-femoral
4- peroneal

after THR surgery by 5 days the patient suffered from severe pain in the thigh and calf
ms ,redness and fever and diffuse edema in the leg but with normal walking the
diagnosis is /
a. infection
b. DVT
c. dislocation of the joint
(b)

causes of failure of stem


break of pmma
poly ethyline failure
large stem

THA in ankylosing spodylitis mostly leat to


1- infection
2- dislocation
3-periprosthetic fracture

X ray In 66 Patient With Union Of Hip And Acetabulam As One Mass Complains Of
Back Pain, Gluteus Medius Not Act So:
. Leave It &Ttt Of Pain
. Constrained Tha

42
.Arthrodesis
. THAWith Metal Prosthesis

After THR surgery by 5 days and on follow up x-ray there was small island of
heterotropic bone around the joint in asymptomatic patient the treatment is
a. radiotherapy
b. open surgery and removal
c. indomethacin 75mg /day
d, reassurance and observation
(c )

HLUMPE Linked Compared To Concentional One :


.High Tolerate To Heat
. High Wear Rate
. Less Tension Fatigue Strength @

Most wearing metal:


a - ceramic on ceramic.
b - polyethylen on polyethylen .
c - metal on metal
d - polyethylen on metal . (True answer??)
e - polyethylen on ceramic .

THA e infection after 9 days with fever, disrupting the sutures and seropurulent
discharge
a.Aspiration and culture ‫كالم منعم‬
b. surgical toilet and replacement of the cup if the infection is deep??
c. imperical antibiotics
d.revision total hip

main benfit of using metal- backed tibial component in TKA


1)improve confirmity of articular sureace
2)reduce maximum compressive stress on the underlying cancellous bone
3) increase tensile force on the other condyl when onre is loading
4)decrease the thickness of polyethelen tray

Minimal Line Thickness InTKA To Decrease Wear Is


6-8 Mm
8-10 Mm
10-12 Mm

Which of the following intra-oprative techniqe decrease the need of latral retinacular
release in TKR
1)int rotation of femo component...
2) ext rotation of femoral component...

43
3)int rot of tibial component......
4)lateralization of patellar component

in total knee replacement the proper allignment is:


a. 7degree varus in anatomical axis
b.7 degree valgus in anatomical axis
c. 7 degree varus in mechanical axis
d. neutral anatomical axis
(b??? )

X-ray total knee aseptic loosening in femoral component: some blood test not indicate to
infection .
a- total knee revision ( true answer ) .
b – arthrodesis .

total knee revision 3 incision was done 2 over longuitudinal parallel and one transverse
the new incision should be
the most medial longui.....
the most lateral longui....
the transverse
new incision longui

6- Osteonecrosis:
Sickle cell anemia patient presented with asymptomatic unilateral hip pain, most likely
diagnosis is:
a) Septic arthritis
b) Avascular Necrosis
c) ?
The correct answer is b

91. Avascular necrosis is detected clinically AFTER


a. 3 month
b. 6 month ??
c. 9 month
d. 11 month
e. 15 month

43-What Is Most Sensitive Diagnostic Method In Early A Symptomatic Avn Hip?


Mri @
Ct
Bone Scan
X Ray

??60. 5years old c/o limping in CT there is AVN ttt is:


a. Surgery total hip replacement

44
b. Splinting
c. Physiotherapy

Young adult patient suffers from long standing pain in the wrist joint and on x-ray
examination there was increased density of the lunate bone the diagnosis is?
a. keinbock's dis.
b.kohler dis.
c.frieberg dis
d. sever's dis
(a)

7-Metabolic:
The common deficiency occur in which trace element in total parenteral infusion:
a. calcium
b. barium
c. zinc
d. magnesium
(c)

??protein….. in the bone drived from : collagen ……..

Osteoblast produce
1-ca
2-collagen
3-vit d

bone is formed of collagen type


a. 1 b. 2 c.3 d.4
(a)

adult daily requirement of CA adult and pregnant .


1300 – 1000.????????

Rickets Occur In Which Layer:


.Calcification
.Hypertrophic
. Prolefrative

What is the cause of this case


Serum ca …… normal
Serum phosphate…..decreased
Urine ca…… decreased
Urine phosphate ………. Increased
Alkaline phosphatase …….. increased
family history

a. Adrenal dysfunction

45
b. Hypophosphatemic rickets
c. Nutritional insufficiency
d. Renal rickets

A case of genu valgus with normal calcium level phosphate level decrease Family
history
a. hypophosphatemic rickets
b. nutritional rickets
c. renal rickets
d. neurfibromatosis

12 year Girl E 24 Genu Valgus Deformity Ttt


High Tibia Osteotomy
Distal Femur Osteotomy@
Both Ostetomy

Osteoclasis can be used to:


Correct deformity of the tibia due to rickets. (Correct Answer)
Curette an osteoclastoma.
Correct deformity.
Correct a ricketery rosary.

The synonym for Paget's disease is:


Osteitis fibrosa. caused by hyperparathyroidism
Osteitisproliferans.
Osteitisdeformans.(Correct Answer)
None of the above.

what the treatment of pt complain from icreased in bone turnover ( paget ) : calcitonin

Picture of pelvic x ray what is diagnosis?


a. Normal
b. Paget’s disease
c. spondylitis
d. osteoporosis

Osteomalacia predominantly affects the:


Spine.
Pelvis.
Skull bones.
Metatarsals

46
A case report 23 patient with calcium level 8 alkaline phosphatase elevated with x ray
osteomalacia and bilateral neck fractures
a. nutritional osteomalacia
b. hyperparathyroidism
c. paget disease
d. multiple myeloma

20 year old girl with decrease BMI =16, history of anorexia nervosa comes in clinic
with complaint of multiple fractures, herbones are so fragile that they often break,
What is your diagnosis:
a. Osteoporosis
b. Hypovitaminosis osteopenia
c. Osteogenesis imperfecta
d. Osteomalacia
Osteomalacia predominantly affects the:
Spine. (Correct Answer)
Pelvis.
Skull bones.
Metatarsals.

Most common cause of non-traumatic fracture in osteoporosis:


a. Verterbral fracture.

1. Patient came with osteoporotic thoracic vertebral fracture t


score for vertebra -2.6 z score The hip -1.6 and z score 0.9
b. Osteoporosis
c. Established osteoporosis
d. Normal bone mass

The term "established osteoporosis" includes the presence of a fragility


fracture, which is present in this scenario.

An old man, not known to have any medical illness that Presented with mid back
pain, he's taking only aspirin, Calcium, And multivitamins. He's not taking dairy
products and on examination he has tenderness in the mid back with mild
kyphosis and X-ray show compression Fracture in the vertebra in, levels what is your
Dx??
a. Osteopenia
b. Osteoporosis
c. Osteomalacia

47
T score of bone densitometry = (-3,5) diagnosis is
a. Osteoporosis

Regading compression facture in osteoporotic patients what is true


a) Normal x-ray rules out the diagnosis??
b) serum alkaline phosphatse is normal
c) Vit d deficiency is the cause
d) steriod therapy is recommended

measurement of bone mass in :/


a.forearm
b.Tibia
c. Humerus
d. Hand
male ptn 50 years old with chronic back pain...PXR shows multiple compression #‎s‬ of spine
with kyphosis...next investigation
a) check serum vit D
b) check serum PTH
c) CT scan
d) DEXA??

Best investigation for measuring bone density


a. DEXA scan

Old lady came to clinic as routine visit, she mention decrease


intake of calcium food, doctor suspect osteoporosis, next initial
investigation:
a. DEXA
b. Calcium in serum
c. Thyroid function test
d. Vitamin D
A patient with osteopenia in the femur with increase serum Alkaline phosphatase,
normal serum calcium, normal phosphate, Normal vitamin D, he is treated with:
a. Estrogen receptor modulator
b. Calcium regulator
c. Bisphosphonate

70y male with osteoporosis the T score of bone densometry would be :


a. -3.5
b. -2.5
c. 1

48
d. 2
e. 3.5
70 year old female patient with osteoporosis, what is her T
score:
a. (-2.5)
b. (-1)
c. (1)
d. (2)

42- Osteoporosis depend on:


a. Age
b. Stage
c. Gender
25- Best exercise for increase muscle strength and bone density
a. Weight and resistance training

X2- The most important exogenous risk factor for osteoporosis is:
a. Alcohol intake
b. Age
c. Smoking
d. Lack of exercise
the correct answer is c

48. A 42 year old man with Cushing syndrome and had a fracture,
you should investigate
a. osteomylitis
b. osteoarthritis
c. osteoporosis

77. Most common site of non-traumatic fracture in osteoporotic


pt. is:
a. Head of femur
b. Neck of femur
c. Vertebra
d. Tibia

Osteoporotic patient on vit D supplement suffered from fracture and the surgeons
planned for ORIF operation what is the most correct scenario for the patient:

a. stop vit D before the operation and calcium monitoring after the operation
b. continue vit D before operation and after

49
c. increase the dose of vit D before operation
d. increase the dose of vit D after operation

A patient with osteoporosis complains of back pain. Which of the following about
vertebral compression fractures is most correct:
a. Normal x-ray vertebra excludes the diagnosis (X)
b. Steroid is a beneficial treatment (X)
c. Vitamin D deficiency is the cause (?)

Old lady afraid of Osteoporosis, to avoid the risk, you should advise her to do:
a) Weight bearing exercise
b) ?
c) ?
d) ?
8-Genetic:
Pt with Lt lower limb shortening 2 cm at age 14 yrs
Rt epiphysiodesis now
Rt shortening at skeletal maturity
Shoe
Lt lengthening

The common features of Neurofibromatosis include all, except


Optic glioma
Dumbellneurofibroma
Scoliosis
Periventricular calcifications (Correct Answer)

Marfan syndrome is associated e defect in


1.Elastin
2.fibrillin
3.fibronictin
4.type1 collagen
5.type3 collagen

X-ray tibia &fibula fracture in both bone e repeated fracture


1-oi
2- diaphysealaclasia
3- ehler dense syndrome

Osteogenesis imperfecta is defect in


)Bone
Calcification
Cartilage
Collagen 1

The Most Good Prognosis Of Tibial Bowing:


Ant Lateral

50
Pos Lateral
Ant Medial
Pos Medial

9-Tumours:

* Important analysis in myelodysplasia patient:


. Urine analysis
. Blood analysis
. IGE latax allergy analysis

14 years boy came to your clinic complaining by pain at mid shaft region with running
x-ray show thickness at the anterior cortex of the tibia next step
MRI
CT
Bone scan
CBC

Osteolytic lesion in proximal femur 10y with limping and tenderness in internal rotation
investigation
1-cbc
2-crp
3-mri
4-ct

12 Years Boy With Back Pain Increase At Night Relaifed By Asprin Progresive
Kyphosis, X-ray Shows Osteolytic Nidus On Bone Ttt:
. Increase Aspirin
. Cortisone Injection
. Surgery @

17 male pt osteoid osteoma in the dorsal spine treatment do scoliosis :


surgery

Osteoid osteoma originates from:


Periosteum.
Cortex. (Correct Answer)
Medullary cavity.
All of the above.

young male patient suffers from chronic pain in his leg and swelling in the tibia the pain
is relieved by salcylate and on x-ray examination there was a nidus radiolucent area in
the diaphysis serrounded by sclerosis the diagnosis is /
a. osteoid osteoma

51
b. pajets disease
c. osteo sarcoma
d. osteochondroma
(a)

-tumor in proximal phalanges of middle finger xray is

X-Ray The Hand With Bone Cyst Phalanx The Diagnose


Enchondroma……..

A l l year old boy presented with the complaints of pain in the right arm near the
shoulder. X-ray examination revealed an expansile lytic lesion in the upper third of
humerus. The most likely diagnosis is:
Giant ccll tumour
Unicameral bone cyst (Correct Answer)
Osteochondroma
Paroteal osteosarcoma

x ray subtrochanteric pathological fracture in single osteolytic lesion


In child so treatment :
a- Pinning .
b -Curretage and biobsy .
c –traction .
d – Platting .

??Scenario of femoral neck cyst x-ray and MRI available


1-ABC
2-bone cyst
3- giant cell tumor
I think abc

xray and mri show mass in proximal femur (radiolucent) , male pt 16 complain of pain of lt hip ,
normal gait , tender extreme internal and external rotation mostly the swelling is
( ansurysmal bone cyst ,
osteoclastoma,
osteosarcome ,
mm )

The most common site of enchondroma is


Ribs.
Phalanges, (Correct Answer)
Clavicle.
Sternum

Osteogenic sarcoma usually occurs in the following age group


Below 15 years (Correct Answer)
Between 16 to 25 years

52
Between 26 to 40 years
Between 41 to 60 years

Sun ray appearance of osteosarcoma is because of:


Periosteal reaction (Correct Answer)
Osteonecrosis.
Calcification along vessels
None of the above.
Sunburst appearance is a type of periosteal reaction giving the appearance of a sunburst secondary to an
aggressive periostitis. It should not be confused with the sunburst sign of meningioma vascularity.

The sunburst appearance occurs when the lesion grows too fast and the periosteum does not have enough
time to lay down a new layer and instead the Sharpey's fiber stretch out perpendicular to the bone. It is
frequently associated with osteosarcoma but can also occur with other aggressive bony lesions such as an
Ewing sarcoma or osteoblastic metastases (e.g. prostate, lung or breast cancer).
osteosarcoma is malignant transformation of:
a.osteoidosteoma
b. osteochondroma
c.Pajet's disease
d.giant cell tumor

A 20-year-old male presented because of increasing pain in his left lower thigh. Examination
revealed tender fusiform thickening of the lower end of the femur with a small effusion into the
knee joint. The overlying skin was warm and the seat of dilated veins but movements of the
knee were free and painless. X-ray examination revealed:
Acute osteomyelitis of the lower end of the femur
Brodie's abscess
Bone sarcoma (Correct Answer)
Fibro sarcoma
Ewinge

Which one of the following statements is untrue concerning chondro-sarcoma:


Occurs most often between the ages 20 and 60 yrs
Is always a primary malignant tumor of bone (Correct Answer)
Most commonly affects scapula, pelvis, ribs & sternum
Causes bone expansion and destruction with irregular opacities in the X-ray
Is radio resistant

Bone metastasis can be best evaluated by:


X-ray.
99mTC bone scan. (Correct Answer)
111Indium scan.
Calcium-alkaline phosphatase elevation.

Multiple myeloma tumor cells resemble:


Granulocytes.
Plasma cells. (Correct Answer)
Lymphocytes.
Chondrocytes.

53
most common site of malignant fibrous histocytoma?
Femur > humerus

An adamantinoma histologically contains:


Squamous cell rests.
Pallisading cells.
Cells resembling basilar cells.
All of the above. (Correct Answer)

Old pt with 2 years bone pain, lethargy, fatigue, wedding gait,


came with table show high calcium and high phosphorus;
a. osteoporosis
b. osteomalacia
c. paget disease of bone
d. metastases prostate cancer (most likely)
e. paraneoplastic syndrome

scenario by fever, loss of weight, bence jones negative but there is mono nuclear cell
،normal ca, ph
mm
metastasis

case senario : fracture femure , pathological , skull show multiple pucched out lesion
Urine analysis shows m body so ttt:
. Traction & radiotherby
. Traction & chemotheraby
. Ext fexator d chemotheraby
. Curretage & bone graft

Bone metastasis in male commonly arises from cancer of:


Lung.
Prostate. (Correct Answer)
Kidney.
Thyroid.

Osteoplastic bone secondaries commonly arise from cancer of:


Breast.
Lung.
Prostate. (Correct Answer)
Adrenal.

Most common symptoms of soft tissue sarcoma:


a. Paralysis
b. Ongrowing mass
c. Pain

A case of female 14 yrs with trendlenberg gait menarche at 9 years with skin lesions
showed at the figure x-ray with osteoporosis at one hip??

54
a.

10-Neuromuscular disorders:
gait analysis ,computer and physical examination is important for outcome of surgery in
a. DDH
b. cerebral palsy
c. TEV
d.poliomylitis

Cp Child Walking In Cruchs , Ascend Upstairs With Side Support , Need Chair If Long
DistanceWhat Gmcs Classification
Grade 1
Grade 2
Grade 3
Grade 4

Gross Motor Function Classification Scale (GMFCS)


Level I Near normal gross motor function, independent ambulator
Level II Walks independently, but difficulty with uneven surfaces, minimal ability to jump
Level III Walks with assistive devices
Level IV Severely limited walking ability, primary mobility is wheelchair
Level V Nonambulator with global involvment, dependent in all aspects of care

Boy 7 Yeer Old With Cp.. He Has Range Of Abduction Of Hips 30 And Popliteal Angle With
Leg About 100 Degree ( ????) And Both Ankles Are Dorseflxed Just To 10 Degrees And
Other Measurements I Cant Remmeber … He Can Stand Dependent And Walk With
Support And Clim Stairs With Supporting To Stair Shelf… He Under Program Of
Rehabilitation And Physiotherapy… Recent Xray Shows Concentric Reductionof Both Hip
Joint >>> Next Step
1. Continue Pt Program
2. Adductor Tenotomy
3.Posterior Root Rhizotomy
4. Actabular Osteotomy

Cerebral Palsy - General Treatment Spasticity & Deformity


•Nonoperative ◦physical therapy, bracing/orthotics, medications for spasticity ◾spasticity
control ◾Botox (botulinum - A toxin) ◾competitive inhibitor of presynaptic cholinergic
receptors with a finite lifetime (usually lasts 2-3 months)
◾used to maintain joint motion during rapid growth when a child is too young for surgery
◾often injected into gastrocnemius
◾helpful treatment in dynamic contractures; little benefit with static contractures

◾baclofen ◾reduces tone via unknown mechanism ◾thought to act as GABA agonist

55
◾intra-thecal administration is preferred route to avoid cognitive impairment seen with oral
administration (poor bioavailability leads to difficulty with oral dosing)
◾Intra-thecal baclofen tends to be used for non-ambulatory CP (GMFCS IV and V)

•Operative ◦soft tissue procedures/releases ◾indications ◾to improve function in child


from 3-5 years of age with spasticity and voluntary muscle control

◾techniques ◾tenotomies for continuously active muscles (e.g. hip adductor)


◾tendon lengthening for continuously active muscles (e.g. achilles tendon or hamstring)
◾tendon transfers for muscles firing out of phase (e.g. rectus tendon or tibialis posterior)
◾tendon transfers in the upper extremity show the best improvement in function in patients
with voluntary motor control

◦selective dorsal rhizotomy ◾indications ◾ages 4 to 8, ambulatory spastic diplegia, and


a stable gait pattern that is limited by lower extremity spasticity

◾neurosurgical resection of dorsal rootlets that do not show a myographic or clinical


response to stimulation
◾contraindications ◾athetoid CP
◾nonambulatory patients with spastic quadriplegia (associated with significant spinal
deformities)

◾falling out of favor due to limited functional gains and no reduced risk of subsequent
musculoskeletal surgeries

◦bony procedures/deformity correction ◾indications ◾usually performed in later childhood


/ adolescence
◾static contractures, progressive joint breakdown, and certain patterned gait-deterioration
can be treated with combinations of myotendonous unit lengthening, tendon transfers, and
osteotomies

◾SEMLS surgery (Single-Event, Multi-Level Surgery) ◾concept arose to limit multiple


surgeries, anesthetics, and rehabilitation time for children
◾most successful when combined with a thorough gait lab assessment that predicts
improvemenet in function with multiple level surgical interventions
◾simple lengthenings can cause deterioration in gait when other contractures are
"uncovered"; SEMLS management seeks to avoid these iatrogenic complications
◾can be done on bilateral lower extremities in efforts to improve gait

Joint least affected by Neuropathy


Shoulder (Correct Answer)
Hip
Wrist
Elbow

A 40-year-old male presented with a grossly swollen painless left knee. Examination revealed
a flail joint with irregularly thickened bone ends, palpably swollen synovial membrane and

56
marked grating and creaking on passive movement of the joint. The first diagnostic step is:
a. Examination of the nervous system.
b. Serological tests.
c. X-ray examination of the joint.
d. Examination of aspirated synovial fluid.
e. Arthroscopy and synovial biopsy
(a)

24-Arthrogryposis Photo
11- Peripheral nerve injury:
Which of the following nerve injuries produce the deformities?
I. Upper trunkII. Porter's tip hand
II. Ulnar nerve111. Claw hand
III. Axillary nerveIV. Flattening of the shoulder
IV. Radial nerveI. Wrist drop

Tinel Sign In Sunderland Type What?


I..Ii…Iii..Iv…V

deep laceration of brachioradialis in midforearm injure what nerve


1.PIN
2. Sup. Radial
3.AIN
4.Ulnar

Which Of This Pt Not Need Operative Ttt:


Open #
Radial N.Palsy
Flotting Elbow

Which one of the following statement is wrong about Erb's palsy?


Abductors of the shoulder are weak
Upper part of brachial plexus is involved
Supinators are normal (Correct Answer)
Internal rotation of arm

All of the following are tests of nerve irritation except:


a. flip test
b.contralateral nerve strech test.
c. Patrick test
d.Lassegue test
(c)

old age uses crutches may cause injury to:


a.axillary nerve
b.median nerve.
C.radial nerve
d.ulnar nerve

57
(c) crutch palsy

pt with tingling of the little finger, atrophy of the hypothenar, limitation of the neck
movement, X-ray shows degenerative cervicitis, EMG study shows ulnar nerve
compression, what will you do://

a. Surgical decompression
b. Cervical CT scan
c. NSAIDS
d. Physiotherapy

Pt came with deep injury on the wrist site, the nerve that has high risk to be injured will
manifest as?

a. Inability to oppose thumb to the other fingers (median nerve)

Patient came after deep laceration at the anterior part of the wrist:
a. Wrist drop
b. Sensory loss only
c. Claw hand
d. Unable to do thumb opposition
The correct answer is d

1- Radial nerve injury : wrist drop , common with humers injury (humers groove)
2- Unlar nerve injury : claw hand , common with elbow injury
3- Median nerve inury :unable to do thumb opposition , common with wrist injury

Case scenario patient present with carpal tunnel syndrome,


appropriate conservative management is by?
a. Splinting, NSAIDS, and corticosteroid injection

Which nerve is compressed in carpal tunnel syndrome:


Ulnar.
Median. (Correct Answer)
Radial.
All of the above

Scenario of Cts The Diagnosis

1-Nerve Conduction

2- Electromyelogram Of Thenar And Paraspinal Muscles

3-Mri Of Cervical Spine

58
A computer programmer presented with wrist pain and +ve
tinnel test. The splint should be applies in:
a. dorsiflexion position
b. palmarflexion position
c. extension position

Patient complaining of pain along median nerve distribution and positivetinel sign treatment
include casting of both hand in what position
a) Dorsiflexion
b) plantar flexion. Neutral is most effective but extension 30 is functional
c) Extension
d) Adduction
e) Abduction

old patient treated from rheumatoid arthritis for a long period he came to your clinic
suffers from burning pain and numbness in the thumb and the next two fingers the
diagnosis is
a. carpal tunnel syndrome
b. ulnar nerve injury

the most common of drop foot


superficial p
deep p
common p
tibial nerve
Sciatic n
‫ لو مش موجود يبقي‬common peroneal‫لو مش موجود يبقي‬
deep peroneal

which nerve is correctly matched to the injury:


a. Carpal tunnel with long thoracic nerve
b. Wrist drop with ulnar nerve
c. Claw hand with radial nerve
d. Interosseous atrophy with median nerve
e. Tarsal tunnel with tibial nerve

Which of the following is false regarding thoracic outlet syndrome?


1. Neurovascular bundle can be compressed between the scalene muscles due to the
presence of a cervical rib
2. patients may experience paraesthesia over medial aspect of the forearm and hand
3. Symptoms may be worsened by hyperabduction of the shoulder joint
4. C5 and C6 nerve roots are commonly affected (Correct Answer)
5. Surgical resection of the 1st rib can be done to relieve the symptoms

Patient complaining of pain at night when he elevated his

59
arm, tingling on lateral arm side and lateral three fingers, Dx/
a. Brachial plexus neuropathy
b. Shoulder impingement syndrome
c. Brachial artery thrombophlebitis
d. Thoracic outlet problem??

Regional:
13 -The shoulder and pectoral girdle
The Rotator cuff is composed of four of the following muscles except/
Teres minor
Supraspinatus
Infraspinatus
Teres major (Correct Answer)
Subscapularis

Painful arc syndrome is due to


Fracture of greater tubercle of humerus
Chronic supraspinatus tendonitis (Correct Answer)
Subacromial bursitis
All of the above.

Painful arc syndrome is seen in all except:


Complete tear of supraspinatus (Correct Answer)
it greater tuberosity
Subacromial bursitis
Supraspinatus tendonitis

Which movement at shoulder gets restricted when suprspinatous torn?


Flexion
Adduction
Abduction (Correct Answer)
Rotation only.

Shoulder pain most commonly due to


Referred pain due to cardiac ischemia
Infraspinatus muscle injury
In acute cholecystitis
Rotator cuff (Correct Answer)

??X. R(bad quality) shoulder ‫مشفتش فيها حاجه‬Male patient with gradual
increasing shoulder pain related to upper forearmMass Proximal to shoulder joint :

60
Metastasis
.?
.?
?
Gradual painful limitation of shoulder movements in an elderly suggest that the most
probable diagnosis is
Arthritis
Osteoarthritis
Periarthritis (Correct Answer)
Myositis Ossificans
Fracture - dislocation

Patient with DM presented with limited or decreased range of


movement passive and active of all directions of shoulder
a. Frozen shoulder
b. Impingement syndrome
c. Osteoarthritis

Which of the following movements are restricted in Frozen shoulder?


Abduction & Internal rotation
Adduction & external rotation
All range of movements (Correct Answer)
Only abduction

Recurrent shoulder joint dislocation is best treated by:


Physiotherapy
Nicola's operation
Bankart's operation (Correct Answer)
Putti-Platt's operation
Arthrodesis of the joint

Caries sicca is seen in


Hip
Shoulder (Correct Answer)
Knee
None of the above

Sprengel's shoulder is due to deformity


Scapula(Correct Answer)
Humerus
Clavicle
Vertebra

61
In Sprengel's shoulder, the following statements are Correctexcept that :
There is congenital elevation &maldevelopment of scapula.
An ugly prominence in the neck is produced by the superior angle.
A band of fibrocartilage or bone anchors the medial border of the scapula to the spine.
There is no limitation of shoulder movements. Shouldr Abduction is restricted
(Correct Answer)
No ttt is required apart from excision of the supermedial angle of scapula to improve
the appearance.

Sprengel's deformity of scapula is :


Undescended / Elevated scapula. (Correct Answer)
Undescended neck of scapula
Exostosis scapula.
None of the above.

Cleidocranial dysostosis may show:


Wide foramen magnum.
Absence of clavicles.
Coxa vara.
All of the above (Correct Answer)

Absent calvicles are seen in


Cleidocranial dysostosis (Correct Answer)
Achondroplasia
Moriquo's disease
Oliver's disease

A 9 - year old child with high arched palate has shoulders meeting in front of his chest. He
has
Erb's palsy
Cleidocranial dysostosis (Correct Answer)
Chondro-Osteodvstrophy
Cortical hyperostosis

14-elbow:

68. Mother come to you complaining of that her child not use his right arm to take
things from her and he keeps his arm in pronation position and fisted, How you will
solve this orthopedic proplem:

a. Orthopedic referral for possible surgical correction


b. Rapid supination of forearm

62
This is a case of nursemaid’s elbow (radial head subluxation) that is
treated by reduction by flexion and supination.

43- Olecranon Bursitis of the elbow joint caused by:


a. Repeated elbow trauma
b. Autoimmune disease
d. Rupture of bursa

Bursitis of the elbow joint caused by:


a) Elbow trauma
b) Autoimmune disease
c) Staph. Aureus
d) rupture of bursa

The correct answer is a

65. Young adult presented with pain on lateral elbow, tingling of


lateral arm, he plays Squash:
a. Carpal tunnel
b. Tennis elbow (Lateral epicondylitis)

71. Male patient c/o pain in his right elbow, he said that he is
using the hammer a lot in his work diagnosis:
a. Lateral epichondylitis
b. Medial epichondyltitis (golfer’s elbow)

15-Wrist

For pollicization of the index finger in child with hypoplastic thumb we require :
a. stable 1st carpo-metacarpal joint
b.stable 2nd metacharpo-phalyngeal joint
c. functioning interphalyngeal joint of the index finger
d. good thenarms.
(b)

According to Wassel classification the most common type of thumb polydactyly is:
a.type 1
b.type 2
c.type 3
d. type 4
(d) 43% and the least type is type 1 (2%)

Mother complains of sharp pain on radial styloid when

63
carrying her baby. The pain increase with extension of the thumb
against resistance, Finkelstein test was positive, Dx:
a. Osteoarthritis of radial styloid
b. De Quervain Tenosynovitis

Stenosing tenovaginitis commonly affects:


Abductor pollicis (Correct Answer)
Flexor pollicis longus.
Opponens pollicis.
All of the above.

A mother complains of pain when she holds her baby in her


wrist. OE radiostaloid tenderness, pain when extend and abduct
the thumb dx??

Also known as de Quervain syndrome, radial styloid tenosynovitis, de


Quervain's tenosynovitis, mother's wrist, or mommy thumb.

Which nerve is affected in Guyon canal :

a.median
b.ulnar
c.radial
d.tibial

Young adult patient suffers from long standing pain in the wrist joint and on x-ray
examination there was increased density of the lunate bone the diagnosis is ?
a. keinbock's dis.
b.kohler dis.
c.frieberg dis
d. sever's dis
(a)

Milch operation is done for


a) negative ulnar variance
b) positive ulnar variance
c) positive ulnar variance with radial shortening
d)positive ulnar variance with radial lengthening

16- Hand
Boutonniere deformity (usually seen with RA)
64
Patient have trauma of his second distal finger after he fall down on it with
hyperextension of the finger, he present to the clinic with pain, redness and he cannot
flex the distal phalanx. What the diagnosis?

a. Intra-articular fracture of distal phalanx.


b. Extra-articular fracture of proximal phalanx.
c. Osteomyelitis
d. Rupture of flexor digitorum profoundus tendon?
e. Rupture of flexor digitorum superficialis tendon

A patient has a picture of osteoarthritis w PIP joints nodules, these are called:
a. Heberden's nodes
b. Bouchard nodes

50 ys old patient diabetic and smoker with thickness in the palmer facia and flexion of
the little and ring finger and thin subcutaneous fascia treated by
a. subcutaneous fasciactomy
b. partial fasciectomy
c. complete fasciectomy
d. amputation.

-scenario of dupetrian contracture e nodule and ulner cord ttt


amputation, partial faschiotomy, complete, scfascetomy )

loss of function may occur when we do repair in flexor tendon in which area:
a. insertion of FDS
b. from the distal palmer crease to the site of insertion of FDS
c. carpal tunnel
d. proximal to carpal tunnel

30 age women with sharp pain in the index finger, increase with the use of scissors or
nail cut which cause sharp pain at the base of the finger in MCP joint and the finger
become directed downward in (mean flexed DIP) and cause pain when try to extend the
finger?

a. Trigger finger
b. Tendon nodule

65
c. dupetren contracure
d. Mallet finger

Fixed Thumb Flexion Deformity 2 Years Boy:


Observation ??
Surgical Release @
Treatment
Nonoperative : passive extension exercises and observation

indications :not recommended for fixed deformities in older children

technique :passive thumb extension exercises, duration based on clinical response

outcomes :30-60% will resolve spontaneously before the age of 2 years old, <10% will resolve
spontaneously after 2 years old

intermittent extension splinting :

indications :first line of treatment, more successful than observation alone

consider alongside stretching regime

flexible deformity,not recommended with fixed deformity in older children

technique :splints maintain IP joint hyperextension and prevent MCP joint hyperextension,duration
for 6-12 weeks

outcomes :50-60% resolution in all age groups.high drop out rate from therapy

Operative:A1 pulley release

indications :fixed deformity beyond age of 12 months of age,failed conservative treatment

outcomes :65-95% resolution in all age groups

* patient notice flexed thumb of baby after 6 months which not extend passively
.splint
. Amputation
. Fasciotomy
. Observation with spontanous recovery at 2 years

Trigger finger, released in which pully ……

a.A2
b.A1
c.A3
d.A4

What component (pulley) of the flexor tendon sheath is commonly involved in trigger
finger?

66
a. A1
b. A2
c. A4
d. C1

Last thing to repair in finger ambutation /


a. Nerve
b. vein
c. artery
d. bone

female with post traumatic of old unitted distal radius 2 months ago comming now as they
describe ???? by reflex sympathetic dystrophy and inflamed hand and fingers with fingers
stiffness ... the appropriat investigation to diagnos it :
1 bone scan
2 mri (correct)
3 ultrasound
4 X ray

‫ فيه سؤال ع‬how to deal with amputated finger till reaching to hospital of reimplantation ‫حد فاكر‬

‫صيغه السؤال؟‬

 In a plastic bag then in ice bag


 Transport of amputated tissue
o indications
 any salvageable tissue should be transported with the patient to
hospital
o modality
 keep amputated tissue wrapped in moist gauze in lactate ringers
solution
 place in sealed plastic bag and place in ice water (avoid direct ice or
dry ice)
 wrap, cover and compress stump with moistened gauze

What is the definitive treatment of frostbite?

N.B. Treatment is by re-warming the affected limb and debridement of


any necrotic tissue.

17,18- Spine:

67
A patient is complaining of occipital & neck pain DX:
a. Occipital Neuralgia

Female presented complaining of neck pain and occipital headache, no other symptoms;
on X-ray she has cervical spine osteophytes and narrow disks, what is the diagnosis?

a. Cervical spondylosis

78--*Photo* Torticollis

child e ms dystrophy had sciliosis>20 degree best treatment :


a. Surgery
a. Orthosis
b. Physiotherapy
c. Cast jacet

What is true about idiopathic juvenile scoliosis


a.Male develop curve more female
b.Douple curve more
c. most commonly appear as a right main thoracic curve

duchen scoliosis no ambulatory pation 30 degree what to do

1-braction to stop it 2-observation 3-surgery


A patient is asked to face the wall, bend his waist, and let his hands hang down without
support. This test is used as a screening tool for which of the following?

a. Scoliosis
b. Lower limb asymmetry
c. Rectal prolapse

Ossificatio Centre Of Iliac Physis Important In :


. Progresion Of Kyphosis
. Progresion Of Sceliosis
. Prognosis Of Sceleosis Surgery
. Determenation Of Age

…Pt with scoliosis, you need to refer him to the ortho when the

degree is:

68
a. 5
b. 10
c. 15
d. 20

A Milwaukee brace can be used in


sacro-iliac strain
derangement of the teeth
a patient with an above knee amputation
scoliosis (Correct Answer)
fractured clavicle

Scoliosis e Marfan’s syndrome ttt is


Conservative trt by TLSO bracing or milkawe bracing according to level

Child with back pain that wake pt from sleep , So diagnosis


a) Lumber kyphosis
b) Osteoarthritis
c) RA
d) Scoliosis

18 years old boy with back pain investigation to do except:


a. CBC
b. ESR
c. X -ray
d. Bone scan?

30 yr old male with mid cervical pain and tenderness. Fever and malaise 2months ago x
ray lateral view show C5-6 body fusion and excessive osteophytes extending to the
anterior border of vertebral body (I feel it like bony ankylosis fusion of the vertebrae .
No intervertebral disc space appear ( bony ankylosis of the intervertebral disc space).
No sever collapse of vertebral body C5 and C6 maintain their anterior body hight)
diagnosis:
a-TB of cervical spin .
b- pyogenic infection of cervical spine .
c- histocytoma .

In Pott's spine, the disease starts in the:


Intervertebral disk.
Anterior vertebral margin.
Posterior vertebral margin.
Paravertebral soft tissue.

patient scenario pain from T12 -L1 weight loss fatigue for 3 month osteopenia and
reduce disc weight what to do
1-ct needle biobsy
2- open biobsy

69
3-six weeks tubercle drugs
4-surgery with fusion

Scenario of caudaequina syndrome and what is ttttttt Operative


o urgent surgical decompression within 48 hours
 indications
 significant suspicion for CES
 severity of symptoms will increase the urgency of surgical
decompression
 techniques
 diskectomy
 laminectomy
 outcomes
 studies have shown improved outcomes in bowel and bladder
function and resolution of motor and sensory deficits when
decompression performed within 48 hours of the onset of symptoms

Indicated Immediate Disectomy In..


Cauda-Equina

30 years old with back pain radiate the the lateral foot…. :
a- spondilolithes .
b -disc herniation ( true answer ) . L4-5 disc
c- spinal stenosis .

L4-L5 disc prolapse:


a. pain in hip and thigh
b. hyposthesia in knee
c. weak dorsiflexors of big toe
d. fasciulation of calf muscle

exam of spine when there is step by palpation it may be

Acute LBP with pain and paraesthesia in Rt lower limb,, with weakness of big toe
extension...management
a)medications and physiotherapy
b) urgent decompression
c) spinal fusion
d) medications for three weeks then if pain persist go for decompression

posterolateral L4-5 disc prolapse cause compression on ……nerve root.//


a. L4
b. L5

70
c. S1
d. caudaequina
(b).Posterolateral and paracentral affect L 5/foraminsl disc affects L4

Case Senario B No Dorsiflexion Of Big Toe , Loss Of Sensation In Anterolateral Aspect


Of Leg
.L1
.L4
.L5 @

Patient with disc prolapse will have:


a. Loss of ankle jerk
b. Fasciculation of posterior calf muscles.
c. Loss of Dorsiflexion compartment of the foot.
d. Loss of the sensation of the groin and anterior aspect of the thigh.

It depends on the level of prolapse; answer (a) is consistent with prolapse at the level of S1-
S2, while answer (c), for example, is consistent with prolapse at the level of L4-L5.

Old pt complaining of back pain on walking on examination there was stiffness of the
muscle and there was some finding on the X-Ray of spondyloarthropathy best effective
ttt:

a. Physiotherapy

b. NSAID

c. Surgery

Commonest Site Of Degenerative Spondylolithesis:


L4-5

spinal stenosis commonly occure in level


a. L3-4
b.L4-L5
c.L5-S1
d.L2-L3

Patient was presented by back pain relieved by ambulation , what is the best initial
treatment :
A. Steroid injection in the back .
B. Back bracing .
C. Physical therapy .

71
The correct answer is c

patient with spinal stenosis didn't relieve his pain with drugs so he did epidural steroid
2 month ago what next .
a – decompression operation.( true answer) .
b- next steroid injection .
c –rest for 3 weeks .

Case: man with low back pain diagnosed as lumbar stenosis.


MX:
a. Physiotherapy
b. Surgical
c. Biofeedback

19-hip:

Thomas test is used to test

a. Flexion deformity of the hip


b. Adduction deformity of the hip
c. Abduction deformity of the hip
d. Hip stability in all directions

Antalgic hip gait is related to which of the following


Waddling gait
Trendelenberg gait
Painful hip gait (Correct Answer)
Shon leg gait

Causes of a painless limp since infancy includes


Congenital dislocation of hip
Infantile coax vara
Poliomyelitis
All of the above. (Correct Answer)

Waddling gait is due to


Gluteal muscle weakness (Correct Answer)
Paravertebral muscle weakness
Obturator nerve palsy
Adductor muscle weakness

In trendlenburg test the trunk:


a. sway on the affected side and the pelvis drops on the opposite side
b. sway on the opp. Side and the pelvis drops on the aff. Side
c. sway on the aff. Side and the pelvis drops on the aff. Side
d. sway on the opp. Side and the pelvis drops on the opp. Side.

72
(a)??

Trenderburg' s sign can be elicited in all of the


following except :
Congenital dislocation of the hip.
Infantile paralysis of the gluteal muscles.
Coxa vara.
Tuberculous arthritis of the hip joint. (Correct Answer)
Non-united fracture of the femoral neck.

Trendelenburg's sign is used in the diagnosis of:


Varicose veins.
Congenital dislocation of the hip. (Correct Answer)
Carcinoma of the stomach.
Pulmonary embolism.

All of the following statements about a positive Trendelenberg's sign are true, except
It occurs with coxa vara
It occurs with paralysis of hip abductors
If it is present on both the sides, the gait can look normal (Correct Answer)
If it is present on one side, the patient has a lurching gait downwards towards the
unsupported side

Standing apparent shortenning of the lower limb is considered


a. adduction contructure of the hip
b. abduction contracture of the hip
(A)

Concerning the diagnosis of congenital dislocation of the hip at the earliest possible moment
in life all are true except
All obstetricians, midwives and general practitioners can be able to diagnose
congenital dislocation of the hip at birth
The condition is diagnosed at birth by eliciting a 'click' or a 'clunk' from the hip
The sign is known as Barlow's or Von Rosen's Sign
Trendelcnberg's sign is positive (Correct Answer)
Shenton's test is positive

The reported incidence of unstable hips per 1000 at


birth is as much as
0.5. Dydplasia 1:100. Dislocation 1:1000
2-5
8-20 (Correct Answer)
25-30
35-40

Congenital dislocation of hip is more commonly seen in


Caucasians (Correct Answer) native American
Negroes
Japanese
Eskimos

73
Which of the following regarding osteoarthritis (OA) is true?
OA is not a genetically determined disease
OA only affects the elderly population
OA of the knees usually presents with deformity
OA of the hip in the Asian population is usually associated with an underlying cause
like dysplastic hip (Correct Answer)
OA is not seen in patients with rheumatoid arthritis

25- Baby With Extension Albow &Knee ,Bilateral Hip Dislocation ,Scoliosis
Which Corect First :
Ankle
Elbow…
Shoulder…
Hip
CDH is due to/
Large acetabulum
Rotation of femur
Small neck femur
Small femoral head

Ortolani's test is done for


Congenital dislocation hip (Correct Answer)
Dislocation patella
Meniscal injury of knee joint
Penhes' disease

Barlow's sign is related to the diagnosis of


talipes equinus varus
congenital dislocation of the hip (Correct Answer)
ulnar nerve palsy
genu varum
fractured neck of femur

Barlow's test is done for testing


CDH in child
CDH in infancy (Correct Answer)
Femoral neck fracture
Slipped femoral epiphysis

In a newborn child, abduction and internal rotation produces a click sound. It is know as :
Otorolani's sign. (Correct Answer)
Telescoping sign.
Mc Murray's sign.
Lachman's sign.

Congenital bilateral dislocation of hip shows(?? All except)


Waddling gait
Lordosis (Correct Answer)
+ ve Trendelenburg test

74
+ ve von Rosen's sign
pt with congenital hip dislocation :
a. Abducting at flexed hip can causes click (Ortolani test)

The most diagnostic sign of congenital hip dysplasia in the newly borne is :
Widening of the perineum.
Asymmetry of the buttocks.
Ortalani's sign. (Correct Answer)
Limitation of hip abduction with hip and knees flexed to 90°.
Apparent shortening of the thigh with the hips and knees flexed to 90°.

TRUE about congenital hip dislocation:


a. Ortolani test
Ortolani & Barlow tests are used to diagnose DDH in infants < 3 m. of
age.

Which one of these statements is True in diagnosis of congenital hip dislocation in the first
few days of life:
It is impossible to diagnose it.
The sign of telescoping is the best way of diagnosing it.
It is possible to diagnose it by the Van Rosen/Barlow Test. (Correct Answer)
The Trendelenberg test is the most useful.

All of the following are absolute indications for radiological evaluation of pelvis for
congenital dislocation of hip, except
Positive family history
Breech presentation (Correct Answer)
Shortening of limb
Unstable hip

Shenton's line is a sign applicable to:


The detection of shortening of the leg on physical examination.
A radiological feature of the pelvis applied to the diagnosis of congenital dislocation of the
hip. (Correct Answer)
A radiological feature of the lungs applied to the diagnosis of pulmonary vein thrombosis.
A physical sign applied to the diagnosis of adrenal deficiency.

Which of the following is seen in bilateral congenital dislocation of hip?


Waddling Gait.
Shenton's line is broken. (Correct Answer)
Trendelenberg test positive.
Allis test positive

The earliest radiological sign in congenital hip dislocation in infants is :


The small shallow acetabulum.
The hypoplastic femoral head.
The shortened anteverted femoral neck.

75
Distortion of Shenton's line. (Correct Answer)
Displacement of the femoral head from the acetabulum.

In congenital dislocation of the hip (CDH), the pathological changes include the following
except :
Small shallow acetabulum.
Snail flattened femoral head lying outside the acetabulum.
Elongated femoral neck. (Correct Answer)
Thickened adherent joint capsule with an hour-glass constriction.
Shortened hamstrings and adductors.

If an unstable hip is detected at birth the management policy is:


Do nothing and re-examine every six months as only a minority of hips develop into a
persistent dislocation.
Use a splint to keep the hip joint in 45° flexion and adduction.
Use a splint to keep the hip joint in 90° flexion and abduction. (Correct Answer)
Advise operative stabilization.

A 10-year-old male with neglected congenital dislocation of the hip presented because of
increasing pain in the back with limping and fatigue. The appropriate management should
be :
Analgesics and anti-inflammatory drugs.
Raising the heel of the right shoe.
Open reduction with deepening the acetabulum by a shelf procedure.
Colonna's arthroplasty.
Lorenz's bifurcation osteotomy. (Correct Answer)

Redirection osteotomy of the pelvis :


a- chiari .
b- ganz .
c- san diego .

DDH hip 2 years old female Unstable during open reduction ttt
capsuloraphy
acetabuloplasty
shortnenig femoral osteotomy
derotational femoral osteotomy

Shortening of femur in DDH


a. To prevent the avascular necrosis of the head
b. to decrease the internal rotation after splint
c. to prevent coxa vara
d. to prevent muscle fibrosis

76
The clinical features of coxa vara include the following except :
Shortening, addution and eversion of the limb.
Raising of the greater trochanter above Nelaton's line.
Limitation of all movements of the hip. (Correct Answer)
Positive Trendelenburg's sign.
Limping and difficulty in kneeling, riding and separating the legs.

most common cause of hip pain in child is


1.scfe
2.perth.s disease
3.transient synovitis
4.septic arthritis
5.juevenile arthritis

A 2 year old is seen in your office. The parent reports that the
child shows toeing in when walking. On examination, the child
exhibits femoral anteversion. The most appropriate treatment is
a. Reassurance to the parent that the condition usually corrects itself
as the child grows older
b. Referral to an orthopedist
c. Referral to a physical therapist
d. Bracing to correct internal rotation of the femurs
e. Fitting for corrective shoes

In the child with Leg-Calve-Perthes Disease (LCPD), which statement is TRUE?


A It is a form of capital femoral epiphysis avascular necrosis secondary to
trauma
B CT imaging is preferred above plain ‘frog leg’ XR’s of the hips
C Patients with suspected LCPD disease require admission for further
inpatient management.
D Is clinically indistinguishable from ‘acute synovitis’

A 7-year-old child presented with intermittent limp and pain in the right hip and knee.
On examination, flexion and extension movements were free and there was no
tenderness and no muscle wasting. X-ray examination confirmed the diagnosis of:
Early tuberculous arthritis of the hip joint
Traumatic arthritis
Perthes' disease (Correct Answer)
Slipped upper femoral epiphysis
Coxa vara

Perthes' disease is common to age group of:


1-5.
6-10. (Correct Answer) 4-8
11-15
16-20.

Which is true about Perthe's disease


Not Painful.

77
It manifests at puberty. (Correct Answer)
Involves neck of femur.
Viral etiology.

Which of the following is true regarding Perth’s disease:/


a. Commonly seen between 11-16 years of age
b. Always unilateral
c. May present by painless limp
d. Characteristically affect the external rotation of hip
e. More in female

4 ys child with perthe's dis. +irritable hip + adductor contructure managed by


a.skin traction
b. adduction brace
c. cheilectomy
d. femoral osteotomy
(a)

Legg-Calve-Perthe's disease is
osteochondritis of the spine
tuberculosis of the hip joint
slipped proximal femoral epiphysis
osteochondritis of the proximal femoral epiphysis (Correct Answer)
osteomalacia

The average duration of Perthes' disease is:


1-2 years. (Correct Answer)
3- 4 years.
1 month - 6 months.
6 months - 1 year.

In Perthes' disease the hip movements restricted are:


Abduction and external rotation.
Abduction and internal rotation. (Correct Answer)
Adduction and external rotation.
All of the above.

obese child pt bik accident , restriction of internal rotation :


SCFE .

in chronic SCFE the best treatment is


a.Closed reduction and fixation
b. open reduction and fixation
c. fixation in situ
d. leave it

78
(c)

The essential examination of the hip in order to clinch the diagnosis of chronic slipped
femoral epiphysis is:
Measuring for shortening of the leg.
Palpation of the femoral head.
A-P plain x-ray view of the hip.
Lateral x-ray view of the hip. (Correct Answer)

The most common site for acute osteomyelitis in infants is


Radial
Femur
Hip joint
Tibia (Correct Answer)

Septic arthritis of infancy usually affects which of the following joints:


Shoulder
Elbow
Wrist
Hip (Correct Answer)
Knee

A psoas abscess present on the right side, the correct statement is


Opposite hip flexion relieves pain
Same side hip flexion relieves pain (Correct Answer)
Same side extension relieves pain
Kyphosis may occur

All of the following movements of the hip are painful in a patient with psoas abscess,
except
Extension
Adduction to abduction
Abduction to adduction
Fixed flexion to further full flexion (Correct Answer)

The most common cause of Bony ankylosis at hip joint


Rheumatoid arthritis
Septic arthritis of hip
Tubercular arthritis
Osteitis deformans

A five year old child is suffering from painful restriction of all movements of hip joint,
the most likely cause in our country is
Congenital coax vara
Tuberculosis arthritis (Correct Answer)
Perthes' disease
Sequelae of septic arthritis of infancy
Psoas abscess

Wandering acetabulum is seen in

79
Fracture of acetabulum
Dislocation of femur
Congenital dislocation of hip
Tuberculosis of hip (Correct Answer)

The most common site of skeletal tuberculosis is


Hip + Spine (Correct Answer)
Knee + Hip joints
Knee joint
Cervical spine

Frame knee in TB hip joint in children is:


a.premature closure of distal femoral physis due to cast for more than a year
b.knee stiffness due to plaster
c. knee stiffness due to muscle fibrosis
d.knee stiffness due to knee infection

A 9-year-old boy developed an intermittent limp which soon became constant and
associated with pain in the Rt hip & knee. Examination revealed a flexion deformity of
the Rt hip with limitation of flexion and extension movements, wasting of the thigh
muscles and upward tilting of the pelvis. The most probable diagnosis is :
Congenital dislocation of the hip.
Legg-Perthes' disease.
Septic arthritis of infancy.
Tuberculous arthritis. (Correct Answer)
Slipped upper femoral epiphysis.

The X-ray findings in tuberculosis of the hip include the following except :
Diffuse decalcification of the bones.
Blurring of the joint outline.
Diminution of the joint space.
Wandering acetabulum.
Downward tilting of the pelvis.(OK) flexion deformity= upward tilting of the pelvis

In children, the treatment of tuberculosis of the hip includes the following except :
Tuberculostatic drugs.
Weight traction to correct deformity.
Fixation of the joint in the position of function.
Aspiration of cold abscess.
Extra-articular arthrodesis. (Correct Answer)

Tertiary syphilitic arthritis most frequently involves The commonest joints to be affected
are the knee, ankle, elbow and shoulder, but smaller ones, such as the interphalangeal joints, are
occasionally affected. There is considerable but painless effusion, but exceptionally, pain may be
severe, and the disease simulates tuberculosis

Spine
Hip
Ankle
Knee

80
20- knee:
arthroscopy knee defect of cartilage less than 2cm tttt by(osteochondoral auto graft,
osteochondoral allograft, chondrocyte implantation )

In 13 y – o – boy, having growth spurt, Dx tibial tubercle pain??


a. Osgood-Schlatter disease
b. stress fracture

Inverted pivot shift test used to detect


a.posterolateral instability
b.anteromedial instability
c.lateral instability
d.medial instability

The normal notch index is


a.0.231
b. 0.312
c. 0.132
d. 0.321

Football player injured in the lateral side of his left knee,


presented to you with sever knee pain, PE: there is swelling in
the medial aspect of the knee, valgus test showed free mobility
but Lachman test and McMurray's test are negative. What’s your
diagnosis?
a. Lateral collateral ligament injury
b. Medial collateral ligament injury
c. Patellar fracture
d. Medial menisci injury
e. Lateral menisci injury

Medial Stabilization Of Patella During Flexion:


-Medial Patellofemural Lig
– Patellar Tendon
– Patellomeniscal Tendon

photo show lower limb and some angles , and need valgus angle of the knee .

-photo for exam knee (acl, pcl and plc, collateral ligament )

fracture condyle tibia with meniscus tear , when we can treat the meniscus
: in same time of condyle surgical fixation

81
A +ve Lachman’s test indicate injury in:
a. ACL tear
b. PCL tear
c. meniscus tear
d. medial CL
e. lateralCL

isolated mcl tear tttt


Conserve
immedite reconstruction
delayed recondtruction

after trauma to the knee the patient examined in prone position and there is increased
external rotation of the knee in 30 degree and 90degree flexion thus he has:
Dial test
> 10° ER asymmetry at 30° only consistent with isolated PLC injury
> 10° ER asymmetry at 30° & 90° consistent with PLC and PCL injury

a. PCL injury
b. PCL and posterolateral corner injury
c. meniscal tear
d. ACL injury
(b)

Meniscal repair,which part e good healing /

a. Acute tear with red on red edges


b. Chronic tear with read on white edges
c. Acute tear with read on white edges
d. Chronic tear with read on read edges

Popping in flexion and extension of the knee …..

a. Discoid meniscus
b. ACL
c. PCL
d. Anterolateral tear

+ve apply distraction test diagnosis is Apley Compression Test To test for meniscus injury
SensitivitY 97%, Specificity 87%2

*Meniscal inj
*collateral lig
*acl

82
In knee injuries which of the following is TRUE?
A A common triad of injuries is ACL/MCL and medial meniscus.
B Proximal tib-fib dislocation occurs with a twisting force to the extended
knee.
C Tears of the patellar tendon need repair due to the high stress of
quadriceps contraction.
D 60% of adolescents with a knee haemarthrosis on x-ray will have an
osteochondral fracture

10-year-old boy presents with a mobile, soft, and painless mass over the posteromedial aspect
of his knee. MRI demonstrates a simple cyst. What is the most appropriate next step in
treatment? Topic Review Topic
1. CT with intravenous contrast
2. Bone scan
3. Radical excision
4. Arthroscopic resection
5. Observation

. 21-ankle& foot:

newborn with bilateral talipesequinovarus the treatment should started:


a. immediatily
b. after 6 months
c. after one year
d. after 2 years
(a)

baby 3 months bilat club foot treated , third stage in ponseti maneuver : correction the equinos .

Boy patient with intoeing c/o W shape of lower limb, not


abducted, Dx:
a. Tibial torsion
b. Femoral torsion
c. Metatarsus adductus

The cause of intoeing depends on the age at presentation:


< 18 m. = metatarsus adductus
18 m. - 3 y. = tibial torsion
> 3 y. = femoral torsion

??Case scenario, baby present with unilateral deformity in the


foot appear when it is become the weight bearing is in the other
foot but when it is the weight bearing the deformity disappear,

83
the patient has defect in dorsiflexion of that foot. I think they
are taking about (club foot)
a. orthopedic correction
b. shoe....
c. surgery

The block test is useful to check the mobility of which


Jointpescavus deformity
Tarso metatarsal
Subtalar
Ankle
1stmetatarsophalyngea

scenario of calcaneus dorsiflexion, talus toward sole, navicular dislocate post the case is
Vertical talus

Adult Acquired Pes Planus Affect


Talus -1st Metatrsal Angle

Regarding hallux valgus, the following statements are true except that it:
Consists of outward deviation of the great toe at the metatarso-phalangeal joint.
Is usually due to badly fitting shoes.
Is not progressive (Correct Answer)
Causes hammer-toe deformity in the other toes.
Predisposes to several painful complications.

The causes of hammer-toe include the following except:


Overcrowding of the toes by ill-fitting shoes.
Hallux valgus.
Pes cavus.
Talipes equinus.
Rupture of the extensor expansion. (Correct Answer)

acute injury big toe flexion pip mild extension mtp


Claw Toe Hammer Toe Mallet Toe

84
DIP flexion extension flexion
PIP flexion flexion normal
normal (slight
MTP hyperextension normal
extension)

hammer toe
mallet toe
claw to

In Hallux valgus surgery, the patients who are likely to be most satisfied are:
Those with pain.
Those with hammertoe.
Those with metatarsus primus varus.
Young age. (Correct Answer)

Hallux valgus is associated with all except


An exostosis on the medial side of the head of the first metatarsal (Correct Answer)
A bunion
Osteo arthritis of the metatarsophalangeal joint
Over-riding or under -riding of the second toe by the third.

Hallux valgus means


Outward deviation of great toe (Correct Answer)
Inward deviation of great toe
Outward deviation of fifth toe
Inward deviation of fifth toe

Hallux valgus with 23 DEGREE mild degree in young patient , for Michele procedure ,
the joint must be
1-congruent
2-incongruent
3-subluxate
(1)

85
Young patient with HTN came complaining of high blood pressure and red, tender,
swollen big left toe, tender swollen foot and tender whole left leg. Diagnosis is:
a) Cellulitis
b) Vasculitis
c) Gout Arthritis
d) ??
The correct answer is a , because tender and swollen whole left leg.

Diabetic male on insulin for 10 yrs…. Come to clinic with swollen redness ankle joint
compaling that he feel non stable ankle when walk and inform a two times of previous
ankle sparin. Small 3 fotosafct show severly destructed ankle joint and lower end of
tibia ( approximately no ankle joint appear) ( notice : noooo pain ) … diagnosis
1 neuropathic joint
2 telangectic osteosarcoma
3 septic arthritis
4 ???

as regard ischemic index diabetic foot or amputation is expected to heel when the index
is:
a. 0.6
b. less than 0.5
c. less than 0.4
d. less than 0.3
posteriortibialis acute rupture clinical pic

X Ray Multiple Small Fractures In Insertion Of Calcaneous In A Dancer


Ttt
. Cortison Inj
. Achillis Augmentation
. Excetion
. Nsaid & Physio Theraby @

Sever's disease related to


a. Talus
b. Calcaneaus
c. Navicular
d. lunate

Athlete man came complain of pain in foot while walking examination there is tenderness i
n planter of foot what is DX:
a. Planter fasciitis
b. Halux vagus

86
c. Hallux rigidus

Scenario and want degree of mangeled ischemic severity score

x ray of osteolytic leisoni think m m and need ttttt (excision, radiation and graft, chemotheraby.

Hemophilic arthritis x-ray in knee (osteo sclerosis, chondro sclerosis, cartilage degenerative.
hemophilic arthropathy

 synovitis
 cartilage destruction
 joint deformity
 pseudotumor

Open fracture calcaneus complication: subtalararthrities, DVT.

Lateral malleolus fracture transverse, medial malleolus fracture verticle by (pronaion abduction,
supination abduction, pronation ex rotation, supination ext rotation

Non union deformity of tibia with multi level deformity and shortening type of osteotomy
(oblique, transverse, varus, valgus.

Scenario case of Blount’s deformity and x-ray with history of osteolytic lesion and child 3y tttt
(osteotomy, curettage and graft, chemo, bracing

Moquit osteotomy for (genu recurvatum, genu valgum, gen varum.

 Articular Cartilage Defects of Knee patellaPatellar cartilage


unloading procedures
o Maquet (tibia tubercle anteriorization)
 indicated only for distal pole lesions
 only elevate 1 cm or else risk of skin necrosis
 contraindications
 superior patellar arthrosis (scope before you
perform the surgery)
o Fulkerson alignment surgery (tibia tubercle anteriorization and
medialization
 indications (controversial)
 lateral and distal pole lesions
 increased Q angle
 contraindications
 superior medial patellar arthrosis (scope before
you perform the surgery)
 skeletal immaturity

87
Inv of avn MRI

S C fracture humerus grade3 posteromedial injury (radial, median, ulner) nerve

Ca for adult and pregnant 1000mg=40mmol for adults…..1500mg =60mmol for pregnent

Gyans canal for

Notch index

Scenario decrease ph and increase ca, alkph, pth(renal failure, gene mutation )

X ray for himiarthroplasty and pain after prolonged walking due to (protruded acetab, aseptic
loosing stem, hyperactivity.

Fracture displaced angulated ulna failed fixation now investigation is

Acute pain in low back and transfered to thigh and outpart of foot due to

Degenerative spondyloleithesis site is L4-5

Scoliosis thoracolumber e 25 degree tttttt is conservative

Scenario of calcaneus dorsiflexion, talus toward sole, navicular dislocate post the case is
Congenital Vertical alus
Inlet pelvic view

Odontoid fracture in obese displaced type 2 tttttttt by


Type II Young Halo if no risk factors for nonunion
Surgery if risk factors for nonunion
Type II Elderly Cervical Orthosis if not surgical candidates
Surgery if surgical candidates

Discharge sinus appear after 12 days of THA tttttttt debridement , lavage, removal of polyeth if
infection deep

-46interlocking tibia from 3 month now there is discharge from proximal locking screw ,xray
screw in its place and bone uniting ttt (remove nail, remove and ex fix, remove and cast, wait
until united then remove.

-ttttt of tardy ulner palsy

88
xray for ankle and there is pain around ankle that wake him from sleep due to
wavy bone un mature, histocyteinfilteration, mononucler cell, polymorphonucler cell

vastusmedialis oblique muscle stabilize knee in flexion e which dynami

saddle anaethesia e urinary incompetence in disc prolapse due to Cauda Equina

-x ray of tibial plateau type3 and what is cause of oA in future(ligament injury, articular
depression, malaligment.

-ttttttt of Paget’s disease bisphosphonate,, calcitonin

-club foot best time ttttt Immediately

scenario case of child 9y pain, swelling, redness in the leg ,no history of injury, ESR ,CRP is
increased what is the case osteomyelitis

Thomas test. evaluates hip flexion contractures

Thompson test. lack of plantar flexion when calf is squeezed in TendoAchilis rupture

sceario case of septic arthritis investigation

autogenus chondrocyte implantation


Operative ◦debridement/chondroplasty vs. reconstruction techniques ◾indications ◾failure of
nonoperative management, ◾acute osteochondral fractures resulting in full-thickness loss of cartilage
◾technique
◾treatment is individualized, there is no one best technique for all defects
◾decision-making algorithm is based on several factors
◾patient facto: age, skeletal maturity, low vs. high demand activities, ability to tolerate extended
rehabilitation
◾defect factors: size of defect, location, contained vs. uncontained, presence or absence of
subchondral bone involvement
◾basic algorithm (may vary depending on published data)

femoral condyle defect: correct malaligment, ligament instability, meniscal deficiency


◾measure size: < 4 cm2 = microfracture or osteochondral autograft transfer (pallative if older/low
demand)
◾> 4 cm2 = osteochondral allograft transplantation or autologous chondrocyte implantation
◾patellofemoral defect ◾address patellofemoral maltracking and malalignment
◾measure size ◾< 4 cm2 = microfracture or osteochondral autograft transfer

89
◾> 4 cm2 = autologous chondrocyte implantation (microfracture if older/low demand)

-63contra indicaion of unicompartmental knee Arthroplasty


Indications

o controversial and vary widely


o as an alternative to total knee arthroplasty or osteotomy for unicompartmental
disease
o classicaly reserved for older (>60), lower-demand, and thin (<82 kg) patients
 6% of patient's meet the above criteria with no contraindications
o new effort to expand indications to include younger patients and patients with more
moderate arthrosis
 Contraindications
o inflammatory arthritis
o ACL deficiency
 absolute contraindication for mobile-bearing UKA and lateral UKA
 controversial for medial fixed-bearing
o fixed varus deformity > 10 degrees
o fixed valgus deformity >5 degrees
o restricted motion
 arc of motion < 90°
 flexion contracture of > 5-10°
o previous meniscectomy in other compartment
o tricompartmental arthritis (diffuse or global pain)
o younger high activity patients and heavy laborers
o overweight patients (> 82 kg)
o grade IV patellofemoral chondrosis (anterior knee pain)

64 -arthrodesis of THA o optimal position


optimal positioning for function and limited effect on adjacent joints
20-35° of flexion
0°-5° adduction
5-10° external rotation
avoid abduction as it creates pelvic obliquity and increased back pain

-71xray tibia, fibula e fracture in both bone e repeated fracture(oI, diaphysealaclasia, ehler
dense syndrome )

-72scenario of OS extend to soft tissue and high grade sarcoma which grade(1A,1B,2A,2B,3)

Stage Grade Site (1) Metastasis


IA Low Grade T1 - intracompartmental M0 (none)
IB Low Grade T2 - extracompartmental M0 (none)
IIA High Grade T1 - intracompartmental M0 (none)
IIB High Grade T2 - extracompartmental M0 (none)
III Metastatic T1 - intracompartmental M1 (regional or distant)
III Metastatic T2 - extracompartmental M1 (regional or distant)

90
-74xray of THA e past history of ttt psoriatic arthrities which increase risk for(heterotrophic, dvt,
dislocation )

-78 ThA 1y ago e recurrent dislocation ttt by reduction now there is soft tissue ttttt by(revison,
remove polyetheline, use constrained socket )

-photo appear leg e bulle and bad condition of skin in the distal after open fixation of pilon
fracture cause of bulle(immediate surgery, tornique )

83 -acase scenario of AS invest(Hlab27,Doble strand DNA, HLADR3)

-84 Nerve injury due to protruded acetablum..........


-85the best responce of part of bone to union is(cortex, BM, periosteum )

-86interlocking femur and fall down on xray appear fracture of mid shaft femur ttttt is(plate e
graft, reaming e big diameter nail )

-87xray of fracture femure in child 6y osteopetrosistttttttt .fixationORIF.............

-88scoliosis e marfan syndrome ttttt Nonoperative

obracing
 indications
 early treatment of mild curve
 outcomes
 less effective than for idiopathic scoliosis
 Operative
o PSF +/- ASF with instrumentation
 indications
 rapidly progressing curve in a skeletally immature
patient
 large curve in a skeletally mature patient
 Extend construct to avoid "adding-on", fuse to
pelvis for distal curves with pelvic oblquity or poor
distal fixation
 Growing rod construct may be required in younger
patients
 Obtain MRI to identify dural ectasia prior to surgery
 outcomes
 higher complication rate than idiopathic scoliosis
surgery

91
-89patient e THA since 1y cannot walk only stand what is the cause(abductor ms denervation,
abductor lever arm failure, acetablum and head not suitable )

-most common complication of THA.....DVT......

-91lag screw is best used e(dcb-compression plate-neutral plate-locking plate)

-92in which type of gutillo used flab(1,2,3a,3b)

-93shortening varus femoral deformity in DDH aimed to... . open reduction and
femoral osteotomy

 indications
 DDH > 2 yr with residual hip dysplasia
 anatomic changes on femoral side (e.g., femoral
anteversion, coxa valga)
 femoral head should be congruently reduced with
satisfactory ROM, and reasonable femoral
sphericity
 best in younger children (< 4 yr)
 after 4 yr, pelvic osteotomies are utilized

............

-flab of tendoachilis in the distal 1/3 of leg from(m gastrocnemius, l gastrocnemius, soleus,
plantaris)

Gluteos medius extensor halluces longus extensor digitorum weakness sensory


disturbance....the root is L4. L5. S1. S2

The most way to stabilize open fracture with transitional rotational and shortning lower limb
fractures.
Ring fixator. Unilateral ex-fix. Interlocking nail biplantar ex fix

X-ray total hip with protruce acetabulum the treatment is


Multiflag with cage. Hemispheric anorhe 2 choice

postermedial approach arthroscopy to repair acl pcl plc

92
scenario anterior drawer test and test for mcl what is it acl+plcacl+mclplc+pcl

osteoidosteoma 2 questions one for diagnose x-ray and the other is treatment

x-ray knee with mild valgus fluorosis hypervitaminosisA scurvy rickets

basocervical 10 years old femoral neck the treatment is traction spica orif crif

the most common of drop foot superficial deep common tibial nerve

1) total knee loose flextion gap what to do Balanced in Extension, Loose in Flexion
Problem: Cut too much posterior femur.
Solution:1) Increase size of femoral component (AP only)
2) Posteriorize femoral component (augment posterior femur).

osteopetrosis child 7 years old with femur fracture treatment skeletal traction spica orif ex-fix

haglund Achilles insertion initial treatment steroid injection stretching calcaneus exostomy
augment with FHL(there is x-ray)

after u put lag screw in the fracture what is the plate u use neutralizing gliding dcp compression

cubitus varus of the elbow how we assess lateral x-ray AP x-ray with bauman angle

x-ray radial prostheses what is the best outcome 1-no more 1 mm in relation with olecranon 2-
use larger head then normal 3-use head with neck prosthes 4-silicone rubber prosthes

scenario total hip in the day 9 start of redness erythema deep squeeze 30 ml of seroguenos
discharge and gram stain positive germ what to do
1-antibiotic for 6 weeks 2-irrigation debridment and leave the wound open 3-wash toilet and
exchang line and head if the infection is deep

clinical picture of medial plate pilon fracture with the wound is long and there is edema and
plasters so whats the cause (they operated the fracture immediately) 1-effect tourniquet 2-
immediate surgery compartment syndrome

patient with compartment syndrome BP 120\80 when we do decompression 30 40 20 10

x-ray of pelvic fracture with ex-fix and plates of the right acetabulum and iliosacral left screw on
the day sex the pation got ill after he was stable bp 130\80 38.6 degree 25 breath time what is

93
the cause 1-infection 2-thrombosis vein

antegrade nail distal femur fracture what is the short distance between the fracture site and the
proximal of the two distal screw to get stable fixation 1-2cm 2-4cm 3-6cm 4-8cm

THA when we do advancement of the greater trochanteric nonunion percent is 1-5% 2-10% 3-
20% 4-30%

disadvantage of the IM nail of the humerus comparing with plate is 1-lower reoperation rate 2-
higher union 3-iatrogenic communition of the fracture4-elbow pain

the most common of scoliosis type 1-idiopathec 2-infantile 3- juvenile 4-adoloscent

duchen scoliosis no ambulatory pation 30 degree what to do1-braction to stop it 2-observation 3-


surgery
duschen patient scoliosis 22 degee develop few degree 14 years old patint what to do 1- leave it
and reassure after 6 month 2-bracing 3- surgery

the enzyme of osteoclast 1- ALP 2- succinate ...

3-the most common site of metastatic 1-thoracolumbar spine 2-proximal femur 3-cervical spine

myelopathy should be assessed for 1-hyperthermia 2-latex allergy

94

You might also like